Vous êtes sur la page 1sur 114

Dr

_F

American Academy of Pediatrics

aq

PREP 2014

eh

Item 2
You are seeing a 16-year-old girl for her annual health supervision visit. The girl has
become a vegan, and her mother is concerned about possible nutritional deficiencies. The
girl has no symptoms and has not lost any weight in the last 6 months. Her menses are
regular and her physical examination findings are unremarkable.
Of the following, the girl's diet puts her at GREATEST risk of a deficiency of
A. calcium
B. iron
C. vitamin A
D. vitamin B12
E. vitamin D

American academy of pediatrics

Dr

_F

American Academy of Pediatrics

aq

PREP 2014

eh

Item 2
Preferred Response:
D
As a vegan, the girl described in the vignette is most likely to develop a deficiency in
B12, a vitamin almost exclusively found in animal products. Vegans require supplements
or fortified food products to meet their need for this vitamin. Because the high intake of
folic acid in vegan diets may mask the hematologic aspects of vitamin B12 deficiency,
diagnosis may be delayed until neurologic symptoms occur. Other potential deficiencies
associated with a vegan diet include calcium, iron, zinc, vitamin A and D, and perhaps
other trace elements, but adequate intake can be ensured by following recommended
vegan food group allowances and serving sizes.
Adolescents may be vegetarian to restrict their caloric intake. Although there are
nutritional advantages to a vegetarian diet, such as decreased intake of protein, saturated
fat, and cholesterol, along with higher amounts of fiber, magnesium, and vitamins C and
E; vegans (those who eat no animal products, ie, meat, poultry, fish, seafood, eggs, and
milk) are at risk for energy deficits and nutrient deficiencies.
Adolescents' nutritional needs are influenced by a number of factors. Biologic
considerations include enhanced needs related to an increased growth rate and change in
body composition; males have a greater blood volume and leaner body mass than
females, who need a minimum fat mass for menstruation and reproduction. In addition to
increased energy needs, growth in lean body mass will require increased intake of
calcium, iron, nitrogen, zinc, magnesium, fluoride, and vitamins A, C, and D. Acute and
chronic illness, trauma, and stress will add to the needs imposed by growth and physical
activity.
Behavioral considerations during the adolescent years include changes in food habits
related to levels of activity, changes in schedules, and a desire for independence, resulting
in skipped meals, more frequent snacking, and more meals outside the home (including
fast food). Last, dissatisfaction with body image may be associated with frequent dieting.
Because boys usually eat more food than girls, their risk of deficiencies is reduced.
PREP Pearls

Adolescents' eating habits put them at risk for nutritional deficiencies.

Vegan diets pose potentially more serious health risks (deficiency of vitamin
B12 but to a lesser extent vitamins A and D and calcium, iron, and zinc)
compared with vegetarian diets.

Vegans/vegetarians may have an underlying eating disorder.


American Board of Pediatrics Content Specification(s):

Understand the potential nutritional deficiencies in adolescents

American academy of pediatrics

Dr

_F

American Academy of Pediatrics

aq

PREP 2014

eh

Suggested Reading:

Adolescent nutrition. In: Pediatric Nutrition Handbook. 6th ed. Elk Grove
Village, IL: American Academy of Pediatrics; 2009:175-181

Nutritional aspects of vegetarian diets. In: Pediatric Nutrition Handbook. 6th


ed. Elk Grove Village, IL: American Academy of Pediatrics; 2009:201-224

Larson N, Neumark-Sztainer D. Adolescent nutrition. Pediatr Rev.


2009;30:494-496. doi:10.1542/pir.30-12-494

Renda M, Fischer P. Vegetarian diets in children and adolescents. Pediatr


Rev. 2009;30:el-e8. doi:10.1542/pir.30-1-el

Messina V, Melina V, Mangels AR. A new food guide for North American
vegetarians. J Am Diet Assoc. 2003;103:771-775. doi:10.1053/
jada.2003.50141

American academy of pediatrics

Dr

_F

American Academy of Pediatrics

aq

PREP 2014

eh

Item 16
A 19-month-old boy is seen in your office for follow-up after a diagnosis of anemia. One
month ago, iron supplementation was initiated for treatment. At that time, his hemoglobin
was 9.1 g/dL (91 g/L), hematocrit was 28% (0.28), and the mean corpuscular volume was
67 fL. Today, his hemoglobin is 11 g/dL (110 g/L), hematocrit is 33% (0.33), and the
mean corpuscular volume is 71 fL. He eats a varied diet and continues to breastfeed well.
His growth is normal.
Of the following, the BEST plan for management today is to
A.
B.
C.
D.
E.

continue iron supplementation for 3 months


encourage more meat in the child's diet
have the mother take a multivitamin with iron tablet daily
offer reassurance and discontinue the iron supplementation
suggest weaning the child from breastfeeding

American academy of pediatrics

Dr

_F

American Academy of Pediatrics

aq

PREP 2014

eh

Item 16
Preferred Response: A
The most common nutritional deficiency in the United States is iron deficiency. It
primarily affects older infants, young children, adolescent girls after menarche, and
women during their childbearing years. Iron deficiency is the most likely cause of
microcytic hypochromic anemia in a healthy breastfeeding toddler. The American
Academy of Pediatrics Committee on Nutrition recommends that presumptive iron
deficiency anemia be treated with oral (elemental) iron at a dose of 3 to 6 mg/kg per day
for 4 weeks. Most commonly, ferrous sulfate (5 mg of ferrous sulfate = 1 mg of elemental
iron) is used. After 4 weeks of supplemental iron, an increase in the hemoglobin
concentration of more than 1 g/dL (10 g/L) or in the hematocrit of more than 3%, as seen
in the boy described in the vignette, confirms the diagnosis of iron deficiency anemia. If
this improvement does not occur, affirmation of adherence with medication
recommendations, or further evaluation for possible blood loss should occur. Histamine 2
blockers and proton-pump inhibitors may decrease iron absorption by raising gastric pH.
If improvement occurs, as in this case, iron supplementation should continue for at least 2
months after the anemia has corrected to replenish the iron stores.
Anemia represents the most severe end of the iron-deficiency spectrum, which is a
limitation of screening for iron deficiency by routine hemoglobin testing alone. Many
studies have documented that early iron deficiency can lead to cognitive and motor
deficits. Some of these deficits are not fully reversible with iron treatment. The potential
for irreversible developmental delay, resulting from a temporary nutritional deficiency,
underscores the importance of prevention, early diagnosis by screening, and complete
treat-ment of iron deficiency.
Increasing iron intake by encouraging more meat in the patient's diet or through daily
multivitamin with iron supplementation in his breastfeeding mother will not supply
adequate additional iron above the daily requirement to replenish iron stores and correct
deficiency. Discontinuing iron supplementation as soon as the anemia corrects may leave
the patient iron deficient and at risk for suboptimal neurocognitive outcomes. Although
breastfed infants need a supplemental source of iron (as either liquid supplement or highiron food) introduced as early as 4 to 6 months of age, discontinuing breastfeeding is not
recommended for the treatment of iron deficiency anemia.
PREP Pearls

Iron deficiency is the most common nutritional deficiency in the United


States.

Presumptive iron deficiency anemia may be treated with oral (elemental) iron
at a dose of 3 to 6 mg/kg per day for 4 weeks.

If improvement in the hemoglobin concentration and hematocrit occurs after 4


weeks of treatment with supplemental iron, iron deficiency is confirmed as the
cause of anemia.

Iron supplementation should continue for at least 2 months after the anemia
has corrected to replenish the iron stores.

American academy of pediatrics

Dr

_F

American Academy of Pediatrics

aq

PREP 2014

eh

American Board of Pediatrics Content Specification:

Know that treatment of iron deficiency with ferrous sulfate should continue
after the hemoglobin concentration has returned to normal
Suggested Reading:
American Academy of Pediatrics. Iron. In: Pediatric Nutrition Handbook. 6th ed.
Elk Grove Village, IL: American Academy of Pediatrics; 2009:403-422
Kett J. Anemia in infancy. Pediatr Rev. 2012;33:186-187. doi:10.1542/pir. 33-4186
Segel G, Hirsh M, Feig S. Managing anemia in pediatric office practice: part 1.
Pediatr Rev. 2002;23:75-84. doi:10.1542/pir.23-3-75
Wu A, Lesperance L, Bernstein H. Screening for iron deficiency. Pediatr Rev.
2002;23:171-178. doi:10.1542/pir.23-5-171

American academy of pediatrics

Dr

_F

American Academy of Pediatrics

aq

PREP 2014

eh

Item 67
A 16-year-old boy is seen for a sports preparticipation physical evaluation. He is an 11th
grader preparing for the wrestling season, and he would like to wrestle at a weight class
that is lower than his current weight. His parents ask you to counsel him about safe
weight control practices during the wrestling season. Upon review of his records, you
note that this teenager has consistently been around the 70th percentile for weight and the
50th percentile for height.
Of the following, the statement MOST appropriate to include in your discussion with this
boy is that
A.
B.
C.
D.
E.

athletes at his age and grade level should not implement a weight loss plan for
sports
an athlete's build does not significantly affect his or her performance, except
for athletes in the extreme ranges for body composition
cycling between higher and lower weights will lead to an increased metabolic
rate and make weight control easier
he should plan his training and weight loss to achieve between 5% and 10%
body fat
mild hypohydration will allow him to safely reduce his weight before events
and will not interfere with performance

American academy of pediatrics

Dr

_F

American Academy of Pediatrics

aq

PREP 2014

eh

Item 67
S
Preferred Response: B
Many young athletes attempt to lose weight in the belief that weight reduction will
enhance their athletic performance. Children and adolescents who participate in activities
that emphasize leanness or a thin physique, such as dance, gymnastics, wrestling, and
distance running, are more likely to attempt to cut weight. For most athletes, build has not
been shown to predict success in sports and other physical activities. However, being
significantly overweight or underweight may be detrimental to an athlete's health and
physical performance. In their policy statement "Promotion of Healthy Weight-Control
Practices in Young Athletes:' the American Academy of Pediatrics (AAP) advises that
children should not implement a weight loss plan for sports before 9th grade. Adolescent
athletes who would like to lose weight should work with their physicians to develop a
gradual weight loss program that incorporates a healthy, diverse diet with adequate
caloric intake. The nutritional program should be paired with an appropriate physical
training regimen that includes strength training and aerobic exercise. Cyclic weight loss
and gain can be an indicator of inappropriate weight loss methods, such as binging and
purging or voluntary dehydration. Dehydration is especially dangerous for young
athletes; children and teens have a higher body surface area to body mass ratio and
decreased sweating capacity and are therefore more susceptible to heat illness.
Additionally, even mild hypohydration has been shown to impair physical performance in
both adults and children.. The use of body mass index to assess weight in relation to
height in children, particularly those with a muscular build, may result in the
inappropriate classification of some young athletes as being overweight. Body
composition measurements combined with height and weight measurements more
accurately reflect body type in young people. Adolescent male athletes should have at
least 7% to 10% body fat. There are no recommendations for minimum body fat in
female adolescents; however, it is generally accepted that female athletes should maintain
a higher percentage of body fat than should male athletes. A body fat content of 14% to
17% is considered "very low" for female athletes. Physicians should counsel young
athletes who want a weight loss program about minimum acceptable weights and healthy
weight control practices.
PREP Pearls

The AAP position statement "Promotion of Healthy Weight-Control Practices


in Young Athletes" provides guidelines for appropriate weight control
measures for child and adolescent athletes.

Unhealthy weight control practices can be especially detrimental to children


and teens.
American Board of Pediatrics Content Specification(s):

Understand the role of fluids in weight control for athletes

Know the appropriate amount of weight loss per week for athletes who
participate in sports with weight categories

Know that many athletes who participate in sports with weight categories
practice weight control that may be pathogenic and pathologic

American academy of pediatrics

Dr

_F

American Academy of Pediatrics

aq

PREP 2014

eh

Suggested Reading:

American Academy of Pediatrics Committee on Sports Medicine and Fitness.


Promotion of healthy weight-control practices in young athletes [published
correction appears in Pediatrics. 2006;117(4):1467]. Pediatrics.
2005;116(6):1557-1564. doi:10.1542/peds.2005-2314

Kiningham RB, Gorenflo DW. Weight loss methods of high school wrestlers.
Med Sci Sports Exerc. 2001;33(5):810-813

Turocy PS, DePalma BF, Florswill CA, et al. National Athletic Trainers'
Association position statement: safe weight loss and maintenance practices in
sport and exercise. J Athl Train. 2011;46(3):322-336

American academy of pediatrics

Dr

_F

American Academy of Pediatrics

aq

PREP 2014

eh

Item 144
A 15-year-old girl fainted at school and is brought to the emergency department. She said
she was in class when she arose from her desk, felt lightheaded, and blacked out.
Presently, she reports feeling well and wants to go home. She did not eat breakfast this
morning. There is no significant past medical history, she takes no medications, and there
is no family history of cardiac problems or epilepsy. On further questioning about her
eating habits, you learn that she has lost 30 pounds in the last 2 months. She reports that
she has been exercising to "tone up" and is eating more "healthy" foods. Her last
menstrual period was 2 months ago, and she is not sexually active. On physical
examination, you note a body mass index of 18, blood pressure of 104/68 mm Hg, and a
heart rate of 48 beats/min. When she stands from a seated position, her heart rate
increases to 78 beats/min and her blood pressure is 90/58 mm Hg. A pregnancy test result
is negative.
Of the following, the MOST appropriate next step in the management of this girl's
condition is to

admit her for further management


advise her to stop exercising for a month
discharge to home after intravenous fluid administration
refer her to a dietitian
refer her to a neurologist

American academy of pediatrics

10

Dr

_F

American Academy of Pediatrics

aq

PREP 2014

eh

Item 144
TE SBP
Preferred Response: A
The girl described in the vignette most likely has an eating disorder and fits the category
of unspecified feeding or eating disorder because her BMI is still between 10-25
percentile for age, and she has not explicitly stated having an intense fear of gaining
weight. She has lost a significant amount of weight in a short period of time, denies the
severity of the situation, and her request to go home may stem from her fear of being
forced to eat and gain weight, if admitted. She meets the medical criteria for admission,
with a resting heart rate of less than 50 beats/min during the day and signs of orthostatic
changes (ie, blood pressure change of >10 mm Hg and pulse rate change of >20
beats/min). Other criteria often used for admission are a heart rate of less than 40/min
during the night; systolic blood pressure less than 90 mm Hg; hypothermia (body
temperature <96F); cardiac arrhythmias; severe malnutrition with a weight less than
75% of ideal body weight for age, sex, and stature; acute weight decrease; refusal of
food; or electrolyte disturbances. All these changes reflect the severity of mal-nutrition
and the need for stabilization. In addition, if there are any concerns about mental health
status (suicidal ideation, a plan, or an attempt) or if the adolescent has been followed up
as an outpatient but is not responding to treatment, she or he should not be sent home.
The immediate goal for admission is to correct metabolic abnormalities and the
malnourished state while being careful to avoid the refeeding syndrome (as a result of
further depletion of already low levels of potassium, phosphate and magnesium as insulin
secretion increases with reintroduction of carbohydrates). The need to correct unhealthy
patterns of eating, promote weight gain, and evaluate for and treat psychiatric
comorbidities should be addressed next along with developing plans for close follow-up
on discharge. This treat-ment plan is best accomplished with a team approach from
medical specialists, mental health personnel, and nutritionists.
Rehydration, if needed for the girl in the vignette, can probably be performed safely by
the oral route. She needs to be placed on bed rest until she is medically stable and
showing consistent weight gain. After discharge it would be advisable to restrict exercise
until continued weight gain is documented because often adolescents will eat while
hospitalized to speed up the discharge but start to restrict soon after. There is no
indication for a neurologic consultation at this time because her syncope is most likely
secondary to postural hypotension.
PREP Pearls

Most adolescents who have eating disorders do not meet all the Diagnostic
and Statistical Manual of Mental Disorders criteria for an anorexia diagnosis
and fall into the Unspecified Feeding or Eating Disorder category.

There are accepted criteria available to guide the need for admission: resting
heart rate less than 50 beats/min during the day and signs of orthostatic
changes; heart rate of less than 40 beats/min during the night; systolic blood
pressure less than 90 mm Hg; hypothermia (body temperature <96F); cardiac
arrhythmias; severe malnutrition with a weight less than 75% of ideal body
weight for age, sex, and stature; acute weight decline; refusal of food; or
electrolyte disturbances.
American academy of pediatrics

11

Dr

_F

American Academy of Pediatrics

aq

PREP 2014

eh

Refeeding syndrome results when phosphorus levels are too low.

American Board of Pediatrics Content Specification(s):

Know the indications for the hospitalization of adolescents with anorexia


nervosa
Suggested Reading:

American Psychiatric Association. Diagnostic and Statistical. Manual of


Mental Disorders Fifth ed. Washington, DC: American Psychiatric
Association; 2013:329-354

Breuner CC. Complementary, holistic, and integrative medicine: eating


disorders. Pediatr Rev. 2010;31:e75-e82. doi:10.1542/pir.31-10-e75

Fisher M. Treatment of eating disorders in children, adolescents, and young


adults. Pediatr Rev. 2006;27:5-16. doi:10.1542/pir.27-1-5

Goldstein MA, Dechant EJ, Beresin EV. Eating disorders. Pediatr Rev.
2011;32:508-521. doi:10.1542/pir.32-12-508

Le Grange D, Doyle PM, Swanson SA, Ludwig K, Glunz C, Kreipe RE.


Calculation of expected body weight in adolescents with eating disorders.
Pediatrics. 2012;129(2):e438-e446. doi:10.1542/peds.2011-1676

American academy of pediatrics

12

Dr

_F

American Academy of Pediatrics

aq

PREP 2014

eh

Item 248
The parents of a 3-week-old, formula-fed infant are concerned about increased fussiness
and spitting up with every feed. The infant is growing and developing normally, has no
skin or respiratory abnormalities, and has had no diarrhea or constipation. They would
like to change the baby from cow milk-based formula to soy formula.
Of the following, the MOST appropriate advice about the proposed formula change is
that soy formula
A.
B.
C.
D.
E.

has been proven effective for the treatment of colic and spitting up
has been proven effective in the prevention of atopic (IgE-mediated) diseases
is beneficial when treating cow-milk enteropathy
provides better support for preterm infant growth
supports term infant growth in a manner equivalent to that of cow milk-based
formula

American academy of pediatrics

13

Dr

_F

American Academy of Pediatrics

aq

PREP 2014

eh

Item 248
I-C
Preferred Response: E
In the United States, approximately 20% of formula-fed infants consume soy formula
despite limited indications for its use. It has a long history of safe use, and multiple
studies demonstrate that soy formula supports normal growth and development in fullterm infants and provides equivalent energy intake to cow milk-based formula. Soy
formula is most appropriately used for infants who have galactosemia or primary lactase
deficiency (extremely rare) who cannot tolerate the carbohydrate source (lactose) in cow
milk formula. Soy formula also is recommended for families who prefer to avoid
consuming animal-derived products (eg, vegan). Soy formula is not an acceptable choice
for preterm infants because soy contains compounds called phytates that bind to calcium,
phosphorus, iron, and zinc. Despite supplementation, these formulas are unable to meet
the calcium and phosphorus needs of preterm infants, thus putting them at risk for
osteopenia. Soy formulas also have markedly higher concentrations of aluminum
compared with human milk, and aluminum competes with calcium for absorption. This
may contribute further to osteopenia in preterm and low birth weight infants and infants
with impaired renal function, but the levels are tolerated by full-term infants with normal
renal function.
Many families and practitioners switch infants from cow milk to soy formula in hopes of
preventing or alleviating various symptoms. Chief among these concerns are colic and
immune-mediated conditions. Controlled trials do not demonstrate relief of colic with soy
formula, but the increased fiber in most soy formula and sucrose in some varieties may be
beneficial. Soy is not indicated for concerns of cow milk intolerance because of high
cross-reactivity. Thirty percent to 64% of infants who demonstrate cow milk protein nonimmunoglobulin E (IgE) enteropathy also experience soy-related enteropathy. A study of
true allergic (IgE) responses revealed a rate of 0.5% for soy formula whereas cow milk
allergy had a rate of 1.8%. In studies of infants with atopic dermatitis, soy allergy was
present in approximately 5% of infants, and the incidence of eczema among infants at
risk for atopic disease was not reduced when breastfeeding was supplemented with soy
formula compared with cow milk formula. Soy formula also has been used for infants
who have presumed transient lactase deficiency secondary to gastroenteritis, but its use
has only limited benefit in decreasing the duration of diarrhea (6 days vs 4 days).
Soy formula contains phytoestrogens, a group of plant-derived nonsteroidal estrogens
that include isoflavones. These compounds closely resemble 17-estradiol, raising
concerns that they will interfere with endocrine function. Epidemiologic and animal
studies are inconclusive and often contradictory. To date no definite evidence is seen of
increased feminization, hypospadias, or major changes in menstrual flow among people
who consumed soy formula as infants. There is concern, based on limited data, that soy
formula may complicate treatment of congenital hypothyroidism. Infants fed soy formula
have a prolonged increase in thyroid-stimulating hormone, and phytates may decrease
uptake of exogenous thyroid hormone in these infants.

American academy of pediatrics

14

Dr

_F

American Academy of Pediatrics

aq

PREP 2014

eh

PREP Pearls

For healthy full-term infants, soy formula supports normal growth and
provides equivalent energy intake compared with cow milk formula.

Soy formula is contraindicated for preterm infants because it does not provide
sufficient calcium and phosphorus to support bone growth.

The most appropriate indications for soy formula are in infants with
galactosemia and congenital lactase deficiency (extremely rare), and when
families desire to avoid all animal products in their diet.

Because of cross-reactivity, soy formula should not be used by infants with


non-IgE-mediated cow milk enteropathy.

There are concerns that phyoestrogens in soy formula could cause endocrine
abnormalities.
American Board of Pediatrics Content Specification(s)

Know the indications for the use of soy formula


Suggested Reading:

Andres A, Cleves MA, Bellando, JB, Pivik RT, Casey PH, Badger TM.
Developmental status of 1-year-old infants fed breast milk, cow's milk
formula or soy formula. Pediatrics. 2012;129(6):1134-1140. doi: 10.1542/
peds.2011-3121

Bhatia J, Greer F, and the Committee on Nutrition. Use of soy protein-based


formulas in infant feeding. Pediatrics. 2008;121(5):1062-1068. doi:
10.1542/peds.2008-0564

Martinez JA, Ballew MP. Infant formulas. Pediatr Rev. 2011;32(5):179-189.


doi: 10.1542/pir.32-5-179

American academy of pediatrics

15

Dr

_F

American Academy of Pediatrics

aq

eh

2013 PREP SA on CD-ROM

Question: 158
A father brings his daughter in for a health supervision visit. She skips (alternating feet) into the room.
You give her a piece of paper and a crayon and ask her to draw a person. She proudly draws a person
with 6 body parts. You ask her to take off her shoes and sweater in order to obtain an accurate weight
and she quickly complies with your request.
Of the following, the age that BEST describes this childs developmental abilities is
A. 2 years old
B. 3 years old
C. 4 years old
D. 5 years old
E. 6 years old

Copyright 2013 American Academy of Pediatrics

Dr

_F

aq

eh

American Academy of Pediatrics

2013 PREP SA on CD-ROM

Critique: 158

Preferred Response: D

The girl described in the vignette has reached the normal motor developmental milestones for a 5year-old. She is able to dress and undress herself, draw a person with 6 body parts, and skip
(alternating feet). A 2-year-old can run well, walk up and down steps one step at a time, jump in place,
make a tower of seven cubes, scribble in a circular pattern, and imitate a horizontal stroke. At 3 years of
age, a child is able to pedal a tricycle, draw a circle, and draw a person with a head and 1 other body
part. A child of 4 years of age is able to copy a cross and a square, catch a bounced ball, and hop on
one foot 2 or 3 times. A 6-year-old child can ride a bicycle without training wheels and draw a person
with details of hair, eyes, nose, mouth, and hands with 5 fingers and who is wearing a shirt, pants, and
shoes.
Parents typically first report delays in motor development when the infant is between 6 and 12
months of age. A childs motor development is assessed using information provided by the parents and
by direct observation. A motor quotient may be determined by dividing the developmental age by the
chronologic age and multiplying the result by 100. Gross motor quotients below 50 are usually indicative
of cerebral palsy. Motor quotients of 50 to 75 may reflect low muscle tone. Children who have milder
motor delays may have comorbid neurodevelopmental disorders, such as learning disorders or attentiondeficit/hyperactivity disorder.
SUGGESTED READING:
Accardo PJ, Capute A. A neurodevelomental perspective on the continuum of developmental
disabilities. In: Accardo PJ. Capute & Accardos Neurodevelopmental Disabilities in Infancy and
Childhood, Volume I: Neurodevelopmental Diagnosis and Treatment. 3rd ed. Baltimore, MD: Paul H
Brookes Publishing Co; 2008:3-10
Blasco P. Motor delays. In: Parker SJ, Zukerman BS, Augustyn MC, eds. Developmental and
Behavioral Pediatrics: A Handbook for Primary Care. 2nd ed. Philadelphia, PA: Lippincott, Williams &
Wilkins; 2005:242-247
Dedrick C. Developmental milestones. Developmental Behavioral Pediatrics Online. 2005
Gerber RJ, Wilks T and Erdie-Lalena C. Developmental milestones: motor developmental. Pediatr
Rev. 2010;31:267-277. doi:10.1542/pir.31-7-267
Schultz M, Blasco P. Motor Development. In: Voight RG, Macias MM, Myers SM, eds. Developmental

Copyright 2013 American Academy of Pediatrics

Dr

_F

American Academy of Pediatrics

aq

eh

2013 PREP SA on CD-ROM

Question: 243
A girl skips (alternating feet) into your examination room for her health supervision visit. She happily
exclaims she is able to tie her shoes and ride her bicycle without training wheels. You give her a blank
piece of paper and ask her to write her name. She proudly writes her first and last name.
Of the following, the age that BEST describes this child is
A. 3 years old
B. 4 years old
C. 5 years old
D. 6 years old
E. 7 years old

Copyright 2013 American Academy of Pediatrics

Dr

_F

aq

eh

American Academy of Pediatrics

2013 PREP SA on CD-ROM

Critique: 243

Preferred Response: D

The child described in the vignette has reached the normal motor developmental milestones for 6
years of age. She can ride a bicycle without training wheels and walk in tandem (skipping). She is also
able to write both her first and last name. A six-year-old should also be able to draw a diamond and draw
a person with 12 to 14 parts.
A 3-year-old is able to copy a circle and draw a person with a head and one other body part. They
also should be able to balance on one foot for 3 seconds and go up stairs alternating feet. A child of 4
years of age is able to copy a square, write part of first name, throw a ball overhand, catch a bounced
ball and hop on one foot 2 or 3 times. Five-year-olds are able to copy a triangle, write their name, hop on
one foot 15 times, and skip. A child of 7 years of age can draw an equal-sided diamond and draw a
person with 18 parts.
Motor developmental milestones are obtained via parental history and from direct observation. To
ascertain if a child has delays in motor milestones, the clinician must take into account history, standard
neurodevelopmental milestones, and the presence or absence of primitive reflexes and postural
responses. A motor quotient (MQ) can be determined by the following: MQ = (motor age/chronological
age) 100. A MQ above 70 is normal, between 50 and 70 is suspicious and requires monitoring, and
below 50 is abnormal and requires referral to a specialist.
SUGGESTED READING:
Blasco P. Motor delays In: Parker SJ, Zukerman BS, Augustyn MC, eds. Developmental and
Behavioral Pediatrics: A Handbook for Primary Care. 2nd ed. Philadelphia, PA: Lippincott, Williams &
Wilkins; 2005:242-247
Dedrick C. Developmental milestones. Developmental Behavioral Pediatrics Online. 2005
Gerber RJ, Wilks T, Erdie-Lalena C. Developmental milestones: motor developmental. Pediatr Rev.
2010;31(7):267-277. doi:10.1542/pir.31-7-267
Perniciaro J. Development, behavior, and mental health. In: Tschudy MM, Arcara KM, ed. The Harriet
Lane Handbook: A Manual for Pediatric House Officers. 19th ed. Philadelphia, PA: Elsevier Mosby;
2012:226-242
Schultz M, Blasco P. Motor development in developmental and behavioral pediatrics. In: Voight RG,
Macias MM, Myers SM, eds. Developmental and Behavioral Pediatrics. Elk Grove Village, IL: American
Academy of Pediatrics; 2011:147-169

Copyright 2013 American Academy of Pediatrics

Dr

_F

American Academy of Pediatrics

aq

eh

2013 PREP SA on CD-ROM

Question: 26
You are caring for a 6-year-old girl who has spastic quadriplegia. She is nonverbal and has severe
motor impairment. Her medical regimen includes sodium valproate, baclofen, and lansoprazole.
Approximately 2 years ago, a gastrostomy tube was placed because her height and weight were both
below the 5th percentile. Since placement of the tube she has experienced no further respiratory
complications and has achieved significant weight gain. Her current nutrient intake includes
approximately 75 kcal per kg per day via gastrostomy plus some pureed foods by mouth. Her mother is
concerned because the child is becoming increasingly difficult to lift from her stroller. At the time of her
office visit today, she appears comfortable and is afebrile. Her height is at the 5th percentile and her
weight is 18 kg, which is at the 25th percentile for her age. The physical examination demonstrates
truncal obesity. Lung auscultation demonstrates clear breath sounds; her abdomen is soft, nontender,
and without masses or organomegaly.
Of the following, you are MOST likely to recommend a dietary regimen that provides a total daily
energy intake of
A. 25 kcal per kg
B. 50 kcal per kg
C. 80 kcal per kg
D. 100 kcal per kg
E. 110 kcal per kg

Copyright 2013 American Academy of Pediatrics

Dr

_F

aq

eh

American Academy of Pediatrics

2013 PREP SA on CD-ROM

Critique: 26

Preferred Response: B

Feeding difficulties have been reported in 30% to 90% of patients who have major motor or cognitive
disabilities, and 90% of nonambulatory children who have cerebral palsy have evidence of mal.
Accordingly, it is important to assess the nutritional status of neurologically impaired children during
health supervision visits. Common causes of poor nutritional status include oral-motor dysphagia,
swallowing dysfunction, gastroesophageal reflux, and aversive feeding behaviors.
The feeding problems of the girl described in the vignette have been successfully managed by
instituting gastrostomy feedings. She currently receives 75 kcal per kg of body weight per day via
gastrostomy, and her feeding regimen provides a total daily intake of approximately 1,350 kcal.
However, over the past 2 years her increased energy consumption has resulted in significant weight gain
and an elevated weight to height ratio.
This childs daily energy intake would be reasonable for a healthy child. However, several studies
have shown that nonambulatory children who have cerebral palsy require only 60% of the daily
recommended energy intake of healthy children. Accordingly, the appropriate caloric intake for the girl
described in the vignette should be reduced to avoid excessive weight gain. Assuming that the energy
requirement of a normal 18 kg child is about 1,400 kcal per day, this childs daily caloric intake should
initially be targeted at 840 kcal per day (1,400 kcal 0.60), or approximately 50 kcal per kg of body
weight per day. Planning a nutritional regimen for the disabled child must consider the underlying
condition, the degree of disability (ambulatory vs nonambulatory, diplegia vs quadriplegia) and the
presence of associated feeding disorders. For example, one study in adolescents and adults indicates
that energy requirements in wheelchair-bound patients who retain spontaneous, purposeful upper
extremity movement approximate that for ambulatory subjects. By contrast, quadriplegic subjects
demonstrated markedly reduced energy requirements. Neurologic disease per se may negatively impact
linear growth, even in the presence of optimal protein-energy consumption. In general, neurologically
impaired children are shorter and weigh less than normal subjects. Undernutrition - defined by weight to
height ratios and skinfold thickness criteria - has been reported in up to 46% of children with cerebral
palsy and in 23% of children with stunted linear growth. However, when adequate energy intake is
provided, often via enteral feedings, weight gain and linear growth may be achieved. In fact, when
feeding disorders are successfully managed, excessive weight gain may become an important clinical
problem, and up to 14% of disabled children have been reported to be obese.
Although an estimate of nutritional status may be calculated with weight-for-height and BMI values,
height and weight are often not recorded in children who have cerebral palsy because of the difficulties
in obtaining measurements. Accordingly, the number of children who become overweight after instituting
enteral feedings is not known. However, it is important to monitor the nutritional status of these children
to assess both their rate of weight gain and their changes in BMI in response to changes in dietary
management. Although the caloric intake of disabled children may need to be reduced to prevent
obesity, it is also important to maintain a daily intake of protein, vitamins, and minerals that matches the
daily recommended intake for healthy children.

Copyright 2013 American Academy of Pediatrics

Dr

_F

American Academy of Pediatrics

aq

eh

2013 PREP SA on CD-ROM

SUGGESTED READING:
Dickerson RN, Brown RO, Hanna DL, Williams JE. Validation of a new method for estimating resting
energy expenditure of non-ambulatory tube-fed patients with severe neurodevelopmental disabilities.
Nutrition. 2002;18(7):578-582. http://www.sciencedirect.com/science/article/pii/S0899900702008067
Kleinman RE, ed. Pediatric Nutrition Handbook. Elk Grove, IL: American Academy of Pediatrics;
2009:821-842
Marchand V, Motil KJ. Nutrition support for neurologically impaired children: a clinical report of the
North American Society for Pediatric Gastroenterology, Hepatology, and Nutrition. J Pediatric
Gastroenterol Nutr. 2006;43(1):123-135
Schwarz SM, Corredor J, Fisher-Medina J, et al. Diagnosis and treatment of feeding disorders in
children with developmental disabilities. Pediatrics. 2001;108(3):671-676. doi:10.1542/peds.108.3.671
Schwarz SM. Feeding disorders in children with developmental disabilities. Infants Young Child.
2003;16(4):317-330
Stallings VA, Zemel BS, Davies JC, et al. Energy expenditure of children and adolescents with severe
disabilities: a cerebral palsy model. Am J Clin Nutr. 1996;64(4):627-634

Copyright 2013 American Academy of Pediatrics

Dr

_F

American Academy of Pediatrics

aq

eh

2013 PREP SA on CD-ROM

Question: 94
An 8-year-old boy is brought to the emergency department because of increasing lethargy. He has a
history of severe cognitive and motor impairment. He is currently living with 5 other children in his third
foster home. His foster mother states that she has been feeding him pureed solids by mouth, but he
chokes and gags when given liquids. His appetite seems to have decreased significantly over the past 2
weeks. She also describes him as severely constipated, requiring frequent suppositories and enemas.
His last bowel movement was 5 days ago. Over the past 24 hours, he has produced only 1 damp diaper.
He appears drowsy, cachectic, and with obvious signs of severe developmental disability. His weight is
17 kg. His respiratory rate is 16 breaths/min and his heart rate is 100 beats/min. Physical examination
shows cracked lips and dry skin with positive tenting. Capillary refill time is 3.5 seconds. The child
demonstrates severe thoracolumbar scoliosis and marked muscle wasting. His extremities are cool and
nonedematous with multiple bruises over the anterior tibial surfaces bilaterally. A large erosion is noted
in the presacral area. The abdomen is soft with mild distension but without apparent tenderness. A large
stool mass is palpated in the left lower quadrant.
Of the following, initial laboratory data will MOST likely demonstrate
A. Sodium: 116 mEq/L
Blood Urea Nitrogen: 8 mg/dL
Albumin: 1.6 g/dL
B. Sodium: 126 mEq/L
Blood Urea Nitrogen: 12 mg/dL
Albumin: 2.8 g/dL
C. Sodium: 135 mEq/L
Blood Urea Nitrogen: 12 mg/dL
Albumin: 4.0 g/dL
D. Sodium: 148 mEq/L
Blood Urea Nitrogen: 26 mg/dL
Albumin: 3.8 g/dL
E. Sodium: 155 mEq/L
Blood Urea Nitrogen: 16 mg/dL
Albumin: 2.0 g/dL

Copyright 2013 American Academy of Pediatrics

Dr

_F

aq

eh

American Academy of Pediatrics

2013 PREP SA on CD-ROM

Critique: 94

Preferred Response: D

The boy described in the vignette is severely disabled and has signs of severe protein-energy
malnutrition and marked dehydration. Protein-energy malnutrition is the most important risk factor for
pediatric morbidity and mortality worldwide. The World Health Organization defines malnutrition as "the
cellular imbalance between supply of nutrients and energy and the body's demand for them to ensure
growth, maintenance, and specific functions." Although a relatively less common problem in the United
States, calorie deprivation leading to marasmus occurs with alarming frequency in chronically disabled
individuals.
The inability to perform daily tasks for self-care and the high prevalence of feeding disorders
(including oropharyngeal dysphagia and gastroesophageal reflux) in severely disabled children
frequently demand alternative feeding techniques via nasogastric or gastrostomy tube infusion. Evidence
of malnutrition has been reported in up to 90% of patients with severe cognitive-motor impairment for a
variety of reasons, including caregiver neglect and failure to recognize feeding disorders. For the boy
described in the vignette, the history indicates overall calorie deprivation, protein deprivation, and
inadequate free water intake. As a result, initial laboratory studies would likely include evidence of
hypernatremic dehydration with modest elevation of blood urea nitrogen. The serum albumin
concentration would be expected to be normal or near normal because of starvation-associated
reductions in basal metabolic rate and reduced nitrogen breakdown and excretion. Because the half-life
of albumin is long, it is an unreliable marker of the malnutrition that is secondary to protein-energy
deficits.
Marasmus is an adaptation to insufficient energy intake and the resulting negative energy balance.
Major clinical risk factors include prolonged hospitalization and clinical conditions, such as cerebral palsy
and intellectual deficiency, cystic fibrosis, celiac disease, malignancy, cardiovascular disorders, endstage renal disease, and cancer. These disorders may lead to decreased energy intake, increased
energy losses, and increased energy expenditure. Adaptation to long-term energy deficits includes
reduced physical activity, a decrease in basal energy metabolism, growth cessation, and weight loss.
These adaptations are an attempt to preserve endogenous protein stores through reduced net
catabolism despite negative nitrogen balance.
Marasmus can be distinguished from kwashiorkor, another form of protein-energy malnutrition, by the
presence of hypoalbuminemia and edema in kwashiorkor. Marasmus is the consequence of inadequate
intake of both protein and calories, while kwashiorkor results from long-term protein deprivation in
association with normal or near-normal energy consumption. These distinct pathophysiological
mechanisms lead to significant clinical differences between the 2 nutritional disorders. Thus, while
inanition may be the only obvious clinical finding in combined protein-energy deficiency (marasmus),
signs of kwashiorkor include evidence of protein deficiency and breakdown, including disruption of
cutaneous and mucosal surfaces, skin and hair hypopigmentation, peripheral edema, and ascites. In
kwashiorkor, dietary consumption of energy in the absence of protein will also lead to marked hepatic
steatosis with or without laboratory evidence of hepatic dysfunction. Considering these clinical and
laboratory characteristics of protein-energy malnutrition, available studies suggest that marasmus
represents the bodys adaptation to starvation, achieved through endogenous protein conservation and
reduced

Copyright 2013 American Academy of Pediatrics

Dr

_F

American Academy of Pediatrics

aq

eh

2013 PREP SA on CD-ROM

metabolic rate, whereas kwashiorkor represents an inability to adapt to starvation characterized by


increased metabolic demand, net catabolism, and signs of protein deficiency. In distinction to marasmus,
kwashiorkor is a rare disorder in the United States, occurring most commonly in elderly patients confined
to nursing homes. In the pediatric population, its incidence is largely limited to children with chronic,
debilitating disease who require prolonged hospitalization.
SUGGESTED READING:
Balint JP. Physical findings in nutritional deficiencies. Pediatr Clin North Am. 1998;45(1):245-60
Blecker U, Mehta DI, Davis R, et al. Nutritional problems in patients who have chronic disease.
Pediatr Rev. Jan 2000;21(1):29-32. doi:10.1542/pir.21-1-29
Day NR, Fried O. Index of suspicion. Pediatr Rev. 2008;29(5):171-176. doi:10.1542/pir.29-5-171
Hendricks KM, Duggan C, Gallagher L, et al. Malnutrition in hospitalized pediatric patients- current
prevalence. Arch Pediatr Adolesc Med. 1995;149(10):1118-1122.
doi:10.1001/archpedi.1995.02170230072010
Joosten KF, Hulst JM. Prevalence of malnutrition in pediatric hospital patients. Curr Opin Pediat.
2008;20(5):590-596. doi:10.1097/MOP.0b013e32830c6ede
Marchand V, Motil KJ. Nutrition support for neurologically impaired children: a clinical report of the
North American Society for Pediatric Gastroenterology, Hepatology, and Nutrition. J Pediatric
Gastroenterol Nutr. 2006;43(1):123-135. doi:10.1097/01.mpg.0000228124.93841.ea
World Health Organization. Nutrition for Health and Development: a global agenda for combating
malnutrition. Geneva, Switzerland: World Health Organization; 2000.
http://www.who.int/mip2001/files/2231/NHDprogressreport2000.pdf

Copyright 2013 American Academy of Pediatrics

Dr

_F

American Academy of Pediatrics

aq

eh

2013 PREP SA on CD-ROM

Question: 95
The mother of a 6-month-old girl in your practice is a vegetarian. She tells you she is planning to wean
her daughter from exclusive breastfeeding over the next few weeks and would like her daughter to also
become a vegetarian. The mother does not eat fish or meat but occasionally consumes eggs and milk.
She asks your advice on how best to pursue a vegetarian diet for her daughter.
Of the following, the statement you are MOST likely to make is
A. high-bulk, high-fiber foods should be offered at every meal
B. infants who consume vegetarian diets are delayed in growth when compared to other infants
C. she should continue to exclusively breastfeed her daughter until the girl is 12 months of age
D. she should include fortified cereals and legumes to ensure adequate protein, vitamins, and minerals
E. vegetarian diets in infants and children are too calorie-restrictive and should be avoided until school
age

Copyright 2013 American Academy of Pediatrics

Dr

_F

aq

eh

American Academy of Pediatrics

2013 PREP SA on CD-ROM

Critique: 95

Preferred Response: D

Parents who follow vegetarian or vegan diets frequently desire the same for their children. It is
important for the provider to identify the type of vegetarianism that parents follow in order to properly
advise them regarding diets for their children. For example, lacto-ovo vegetarians eliminate only meat
from their diets and will eat milk and eggs, whereas vegans eliminate all animal products (including milk
and eggs) from their diet. Parents who choose a vegetarian diet for their children need to ensure that
they receive adequate folate, vitamin B12, and omega-3 fatty acids because these nutrients may be
deficient in such diets.
In addition to ensuring adequate intake of these nutrients, parents should offer their children a wide
variety of foods to provide sufficient protein. Fortified cereals and legumes (beans, peas, and lentils) are
excellent choices. In addition, children should consume a limited quantity of high-bulk, high-fiber foods
because these may cause early satiety and children may stop eating before they have met their
nutritional requirements. Finally, while breastfeeding until at least 12 months of age should be
encouraged, supplemental iron-rich foods should be offered at approximately 6 months of age to avoid
iron deficiency. The children of parents who follow the above guidelines carefully grow at the same rates
as children who consume meat.
Pediatric healthcare providers need to familiarize themselves with the varied and often culturally
based feeding practices that may be used by parents. For example, it is not uncommon for mothers who
are born outside the United States to feed their very young infants liquids or foods such as herbs, sugar
water, and honey, which may increase the risk of electrolyte abnormalities or botulism. Infants who are
fed a diet of strictly goats milk are at risk for folic acid deficiency and resultant anemia. Active
questioning about the use of these foods is the key to recognition and appropriate counseling. The
provider is encouraged to ask general questions in a nonjudgmental fashion, such as How do you plan
to feed your baby? or Do you give your baby anything to eat or drink besides breast milk or formula?
The goal of such discussions is to maintain cultural sensitivity while providing basic nutritional education.
SUGGESTED READING:
Pak-Gorstein S, Haq A, Graham EA. Cultural influences on infant feeding practices. Pediatr Rev.
2009;30(3):e11-e21. doi:10.1542/pir.30-3-e11
Renda M, Fischer P. Vegetarian diets in children and adolescents. Pediatr Rev. 2009;30(1):e1-e8.
doi:10.1542/pir.30-1-e1

Copyright 2013 American Academy of Pediatrics

Dr

_F

American Academy of Pediatrics

aq

eh

2013 PREP SA on CD-ROM

Question: 179
A 16-year-old girl is brought to your office by her parents. They are worried about her excessive weight
loss. At the time of her last office visit 1 year ago, the girl had a weight of 56 kg, a height of 1.6 m, and a
body mass index of approximately 21. Over the past 9 months, she has lost 13 kg through diet and
vigorous exercise. She stopped menstruating 3 months ago. Her current body mass index is 16. She
denies any physical symptoms and is upset that her parents have brought her to the doctor. You
recommend an increase in her daily energy intake and a modification in her exercise regimen with
follow-up in 2 weeks.
When she returns to the office, her weight has decreased 1 kilogram since the last visit and she
appears cachectic on physical examination. Vital signs show a resting heart rate of 50 beats/min and a
blood pressure of 90/60 mm Hg. You elect to admit her to the hospital for nutritional rehabilitation. Upon
admission, she refuses to take any dietary supplements and limits her intake to water, fruits, and
vegetables. After discussing the situation with her distraught parents, you elect to begin nasogastric
feeding, which will initially provide her with 2,500 kcal per day. Three days after commencing this
regimen, she complains of fatigue and weakness in her lower extremities. Laboratory studies are
ordered that demonstrate the following:

White blood cell count, 3,500/L (3.5 109/L)


Hemoglobin, 13.5 g/dL (135 g/L)
Sodium, 135 mEq/L (135 mmol/L)
Potassium, 2.8 mEq/L (2.8 mmol/L)
Chloride, 105 mEq/L (105 mmol/L)
Carbon dioxide, 22 mEq/L (22 mmol/L)
Calcium, 7.5 mEq/L (1.86 mmol/L)
Phosphorus, 2.0 mg/dL (0.64 mmol/dL)
Magnesium, 1.2 mEq/L (0.6 mmol/L)
Glucose, 115 mg/dL (6.3 mmol/dL)

Of the following, you are MOST likely to recommend


A. beginning total parenteral nutrition
B. changing from bolus to continuous enteral feedings
C. decreasing calorie intake to 1,250 kcal per day
D. increasing calorie intake to 3,000 kcal per day
E. supplementing enteral feedings with calcium and electrolytes

Copyright 2013 American Academy of Pediatrics

Dr

_F

aq

eh

American Academy of Pediatrics

2013 PREP SA on CD-ROM

Critique: 179

Preferred Response: C

Most likely, the girl described in the vignette has anorexia nervosa. Her continued weight loss and
physical findings indicate severe malnutrition and she must be hospitalized. She has been started on
enteral feedings via the nasogastric route with a total intake of 2,500 kcal per 24 hours. This level of
energy consumption represents the approximate daily recommended intake for an ambulatory 70 kg
individual (when using the calculation of 100 kcal per kg for the initial 10 kg body weight, 50 kcal per kg
for the second 10 kg, and 20 kcal per kg for each kg of body weight above 20 kg). However, 3 days after
instituting this regimen she develops muscle weakness, fatigue, hypocalcemia, hypokalemia,
hypomagnesemia, and hypophosphatemia. These findings suggest the development of refeeding
syndrome, which is the consequence of overly aggressive nutritional management of the malnourished
patient. Following correction of electrolyte deficits, feedings should be restarted at a rate that provides
about 50% of targeted intake, or 1,250 kcal per day.
When deciding upon an appropriate nutritional support regimen for the hospitalized patient, a simple
but clear axiom should always prevail: If the gut works, use it. Parenteral nutrition should be reserved
only for those situations in which gastrointestinal (GI) motility or absorptive function are abrogated.
However, even under these conditions, the availability of either partially or completely hydrolyzed
formulas has greatly enhanced the ability to provide adequate nutrition via the enteral route.
In general, peripheral venous nutrition is used in patients who are unable to receive oral or enteral
nutrition for periods from 3 to 7 days' duration, and total parenteral nutrition (TPN) is instituted when
patients must receive nothing by mouth for more than 7 days. Although enteral feedings pose fewer
significant risks when compared to parenteral nutrition - including lower risks of infection, fluid and
electrolyte disturbances and catheter malfunction - care must be taken to avoid problems related to
infusion rate and composition. The most common tube-feeding-related complications are listed in (Item
C179). Many of the mechanical tube-related complications have been obviated by the development of
softer feeding tubes (silastic, polyurethane) that pose a far lower risk for perforation or mucosal irritation.
Refeeding syndrome is a well-described phenomenon that develops as a consequence of specific
metabolic events that accompany aggressive energy supplementation. Patients particularly at risk
include those with the following conditions:

Anorexia nervosa

Kwashiorkor or marasmus

Chronic malnutrition

Chronic alcoholism

Prolonged fasting

Prolonged IV hydration

Metabolic stress (eg, chronic disease) and nutrient depletion


During periods of starvation, insulin levels fall and promote the release of glucose and free fatty acids
for energy. Thyroid hormone levels are also reduced and result in a lower metabolic rate; insulinlike
growth factor 1 levels fall and cause a reduction in protein synthesis. When glycogen stores are
exhausted, upregulation of gluconeogenesis from protein catabolism leads to water, vitamin, and mineral

Copyright 2013 American Academy of Pediatrics

10

Dr

_F

American Academy of Pediatrics

aq

eh

2013 PREP SA on CD-ROM

depletion. A sudden, marked increase in carbohydrate intake with refeeding will raise serum insulin and
lower serum glucagon levels. Increased cellular glucose uptake then promotes intracellular movement of
phosphate, potassium, and magnesium, while increased utilization of vitamins (including thiamine) and
adenosine triphosphate lead to a deficiency state. Clinically, multiple organ systems may be affected,
resulting in muscle weakness, seizures, cardiac arrhythmias, hypotension and ileus. To avoid the
refeeding syndrome, nutritional rehabilitation of patients who are at high risk for refeeding syndrome
should proceed as follows:
1.
2.
a.
b.
3.
4.

Correct electrolyte abnormalities.


Institute oral or enteral nutrition.
Start at a calculated intake of approximately 50% targeted energy consumption.
Advance energy and volume intake slowly.
Closely monitor vital signs, intake and output, and serum electrolyte levels.
Provide appropriate vitamin supplementation for age.

The decision to institute bolus versus continuous feedings via the enteral route will depend on the
patients clinical status and gastrointestinal motility. Severely malnourished subjects may benefit from a
slow, continuous infusion rate to avoid problems created by rapid hormonal changes that lead to marked
fluid and electrolyte shifts. Because severe malnutrition is frequently associated with electrolyte
disturbances and alterations in GI motility, including gastroparesis, great care must be taken before
instituting an oral or enteral feeding program. In severely malnourished patients, fluid and electrolyte
abnormalities should be corrected via the intravenous route rather than by adding electrolytes to the
enteral feeding regimen. As stated above, a period of parenteral nutrition may be required in those
situations where GI function is significantly compromised. For the girl described in the vignette, the initial
decision to employ enteral feedings was appropriate, and either the bolus or the continuous route would
have been an acceptable alternative. However, since her gastrointestinal tract has already tolerated
bolus enteral feedings, a significant reduction in energy intake should represent the initial modification in
her nutrition support program.
SUGGESTED READING:
ASPEN Board of Directors and the Clinical Guidelines Task Force. Guidelines for the use of
parenteral and enteral nutrition in adult and pediatric patients. J Parenter Enteral Nutr. 2002;26(suppl
1):1SA-137SA
Fisher M. Treatment of eating disorders in children, adolescents, and young adults. Pediatr Rev.
2006;27(1):5-16. doi:10.1542/pir.27-1-5
Fisher M, Simpser E, Schneider M. Hypophosphatemia secondary to oral refeeding in anorexia
nervosa. Int J Eat Disord. 2000;28(2):181-187.

Copyright 2013 American Academy of Pediatrics

11

Dr

_F

American Academy of Pediatrics

aq

eh

2013 PREP SA on CD-ROM

Fuentebella J, Kerner JA. Refeeding syndrome. Pediatr Clin North Am. 2009;56(5):1201-10.
http://www.ncbi.nlm.nih.gov/pubmed/19931071#
Metheny NA , Mueller C, Robbins S, et al. A.S.P.E.N. enteral nutrition practice recommendations. J
Parenter Enteral Nutr. 2009;33(2);122-167. doi:10.1177/0148607108330314
Solomon SM, Kirby DF. The refeeding syndrome: a review. J Parenter Enteral Nutr. 1990;14(1):90-97

Copyright 2013 American Academy of Pediatrics

12

Dr

_F

aq

eh

Dr

_F

American Academy of Pediatrics

aq

eh

2013 PREP SA on CD-ROM

Question: 180
You are addressing a group of expectant mothers at a local health clinic as part of a community
education program. You explain that exclusive breastfeeding is the best nutrition for most infants after
birth and that introduction of other foods should be delayed for several months.
Of the following, the recommendation regarding the introduction of complementary foods you are
MOST likely to make is
A. exclusive breastfeeding should continue until 12 months of age, followed by introduction of any solid
food
B. iron-containing cereal mixed with formula should be offered via a bottle at age 3 months
C. iron-containing cereal should be offered via a spoon between 4 and 6 months
D. juice and supplemental water should be offered via a bottle at age 2 months
E. once solid food is initiated, several foods should be offered at once to improve acceptance

Copyright 2013 American Academy of Pediatrics

14

Dr

_F

aq

eh

American Academy of Pediatrics

2013 PREP SA on CD-ROM

Critique: 180

Preferred Response: C

Breastfeeding is widely regarded as the most appropriate food for newborn infants, but
complementary foods should be offered no later than 6 months of age in order to provide the optimum
amount of nutrients such as iron. The introduction of solid foods varies widely between cultures, and
there is no known optimum sequence. However, the American Academy of Pediatrics recommends that
infants receive iron-containing cereal between 4 and 6 months of age. This cereal should be offered with
a spoon rather than mixed with formula or human milk in the bottle. Fruits and vegetables may be
offered once the cereal is tolerated, and parents should introduce new foods one at a time every few
days. Repeated introduction of some foods may be necessary as infants develop taste preferences.
Juice should not be given before 6 months of age and should be given in a cup rather than a bottle.
Infants should be allowed to self-regulate their eating, so their intake may vary from meal to meal.
Parents should control when food is available to the infant, and it is important to set aside time for meals
and snacks rather than allowing the infant to eat while playing. There is no difference in growth between
infants who eat commercially prepared baby foods and those who receive foods prepared by their
parents. If possible, breastfeeding should continue for at least 12 months because it may lead to better
self-regulation in later eating habits and has numerous other health benefits.
SUGGESTED READING:
Gidding SS, et al. Endorsed policy statement. Dietary recommendations for children and adolescents:
a guide for practitioners. Pediatrics. 2006;117(2):544-555. doi:10.1542/peds.2005-2374
Grummer-Strawn LM, Scanlon KS, Fein SB. Infant feeding and feeding transitions during the first
year of life. Pediatrics. 2008;122(sup 2):S36-S42. doi:10.1542/peds.2008-1315d
Mehta KC, et al. Trial on timing of introduction of solids and food type on infant growth. Pediatrics.
1998;102(3):569-573. doi:10.1542/peds.102.3.569

Copyright 2013 American Academy of Pediatrics

15

Dr

_F

American Academy of Pediatrics

aq

eh

2013 PREP SA on CD-ROM

Question: 245
A 15-year-old boy presents for his annual health supervision visit. His mother and father accompany
him to the visit and you note that both his parents are overweight. His father also has a history of type 2
diabetes mellitus. On review of his growth records, you note that the boys body mass index is 38. His
physical examination reveals a blood pressure of 135/84 mm Hg and thick acanthosis nigricans on his
neck. You order laboratory tests that reveal the following:
Hemoglobin A1c, 6.7% (0.07)
Thyrotropin, 5.9 mIU/L
Free thyroxine, 1.2 ng/dL (15 pmol/L).
You decide to initiate diet and exercise modifications as well as medication for this boy.
Of the following, the MOST appropriate medication to prescribe for this boy is
A. enalapril
B. glargine
C. glyburide
D. levothyroxine
E. metformin

Copyright 2013 American Academy of Pediatrics

16

Dr

_F

aq

eh

American Academy of Pediatrics

2013 PREP SA on CD-ROM

Critique: 245

Preferred Response: E

The boy described in the vignette is obese, has obese parents, and has a family history of type 2
diabetes mellitus. Parental obesity and obesity in adolescence are strong predictors of obesity in
adulthood. In 2010, the American Diabetes Association approved the use of hemoglobin A1c (HbA1c)
measurement as a primary tool for diagnosing diabetes. The threshold for diagnosing diabetes was set
at an HbA1c level of 6.5% (0.07), with an HbA1c level above 5.7% (0.06) designated as prediabetes.
This boy has an HbA1c level of 6.7% (0.07) and therefore meets revised diagnostic criteria for diabetes.
Current recommendations for the management of new-onset type 2 diabetes include both lifestyle
modification and the initiation of pharmacologic therapy.
Metformin would be the most appropriate medication to prescribe for this boy. Glargine insulin is
typically reserved for the management of patients with HbA1c levels higher than 9% (0.09) or those who
are unable to achieve glycemic control with oral agents. Glyburide is a sulfonylurea and can be used in
the management of type 2 diabetes but is considered a second-line agent to metformin. Enalapril may
eventually be required in this patient, but additional efforts at lifestyle modification (eg, diet and exercise)
would be recommended before initiating an angiotensin-converting enzyme inhibitor. Finally,
levothyroxine would not be recommended because this boy does not have thyroid dysfunction. Although
his thyrotropin concentration is mildly elevated at 5.9 mIU/L, modest elevations in thyrotropin are
commonly seen in obese patients and are not associated with primary thyroid dysfunction. His normal
free thyroxine concentration also suggests that he does not have thyroid dysfunction.
SUGGESTED READING:
McGovern L, Johnson JN, Paulo R, et al. Clinical review: treatment of pediatric obesity: a systematic
review and meta-analysis of randomized trials. J Clin Endocrinol Metab. 2008;93(12):4600-4605.
doi:10.1210/jc.2006-2409
Orsi CM, Hale DE, Lynch JL. Pediatric obesity epidemiology. Curr Opin Endocrinol Diabetes Obes.
2011;18(1):14-22. doi:10.1097/MED.0b013e3283423de1
Taveras EM, Gillman MW, Kleinman K, Rich-Edwards JW, Rifas-Shiman SL. Racial/ethnic
differences in early-life risk factors for childhood obesity. Pediatrics. 2010;125(4):686-695.
doi:10.1542/peds.2009-2100
Stichel H, lAllemand D, Grters A. Thyroid function and obesity in children and adolescents. Horm
Res. 2000;54(1):14-19. doi:10.1159/000063431
Whitaker RC, Wright JA, Pepe MS, Seidel KD, Dietz WH. Predicting obesity in young adulthood from
childhood and parental obesity. N Engl J Med. 1997;337(13):869-873.
http://www.nejm.org/doi/full/10.1056/NEJM199709253371301

Copyright 2013 American Academy of Pediatrics

17

Dr

American Academy of Pediatrics

_F

aq

2012 PREP SA on CD-ROM

Question: 1
You are caring for a 2-month-old infant who has hypoplastic left heart syndrome. He weighed 2,800 g
at term birth. The baby has undergone the first stage of the multistage corrective surgical procedure. His
medications include digoxin and furosemide to ameliorate congestive heart failure. He visited his
cardiologist last week, where his weight was 3,250 g, and no changes in medication dosages were
recommended. At the time of his office visit today, his parents report that the baby is taking 60 mL of a
24 kcal/30 mL cow milk protein-based formula every 4 hours, but he seems to tire easily. He spits up
formula once or twice after each feeding. His weight today is 3,300 g.
Of the following, the MOST appropriate recommendation for feeding this infant is

A. an every-2-hours feeding schedule


B. an amino acid-based formula
C. formula thickened with rice cereal
D. medium-chain triglyceride supplements
E. nasogastric feedings

Copyright 2012 American Academy of Pediatrics

eh

Dr

_F

American Academy of Pediatrics

2012 PREP SA on CD-ROM

Critique: 1

Preferred Response: E

aq

Disturbances in growth and weight gain are common in infants who have congenital heart disease
and congestive heart failure (CHF). Clearly, the most effective preventive measure for such nutritional
complications is total surgical correction of the cardiac lesion. When that option is not feasible,
adjunctive therapy must be directed at providing sufficient energy intake to assure a relatively normal
growth rate, including incremental calorie supplementation to respond to increased metabolic demand
and to achieve catch-up growth, if necessary. Studies have indicated that infants who have CHF and
growth failure require 140 to 150 kcal/kg per day to achieve these objectives.
The weight gain for the infant described in the vignette has been poor (about 7 g/day in the past
week), and his total daily energy intake is approximately 250 kcal/day or less than 80 kcal/kg per day.
The most effective method of dietary management to achieve the targeted intake needed is nasogastric
feedings. Although various formulas (including amino acid-based preparations) and energy
supplementation (rice cereal, medium-chain triglycerides, glucose polymers) may be used, evidence
suggests that the necessary energy intake goal cannot be achieved with oral feedings, either alone or in
combination with nighttime enteral nutrition. More frequent feedings will not aid in achieving the goal.
Energy requirements are highly variable in healthy children and are influenced by basal metabolism,
growth rate, physical activity, sex, body size, and developmental stage. Numerous methodologies have
been proposed to determine total energy requirements for infants, children, and adolescents. Several
recently developed equations for estimating energy needs from birth through adolescence are offered by
Glassman and Kleinman (see references). However, these guidelines do not account for special
situations in which energy needs may increase because of a chronic illness, such as cystic fibrosis, or
decrease because of reduced activity, such as for the child who has cerebral palsy and severe motor
impairment. As demonstrated by this infant, energy requirements may vary widely from established
norms, not only because of increased energy expenditure related to CHF, but also because of the need
to achieve catch-up growth in infants who have long-standing undernutrition. For the malnourished child,
estimates of energy intake necessary to achieve catch-up growth alone may be calculated as follows,
using the National Center for Health Statistics weight for height data:
kcal/kg/day = RDA for weight age (kcal/kg) x target weight for height
actual weight
where the weight age represents the age at which the child's current weight would be at the 50th
percentile and the target weight for height is the median weight for the patient's height. In many cases,
catch-up growth alone demands an energy intake totaling 120% to 125% of the age-related
recommended daily allowance (RDA). Increased needs related to clinical disease states must be added
to this estimate.
Enteral nutrition may be defined as the provision of liquid nutrition that involves complex, partially
hydrolyzed, or elemental diets, generally via a nasally or percutaneously placed feeding tube. Enteral
feedings are an essential component of care for patients who are unable to satisfy their nutritional
requirements through regular oral feedings and may be warranted under the following clinical

Copyright 2012 American Academy of Pediatrics

eh

Dr

American Academy of Pediatrics

_F

aq

2012 PREP SA on CD-ROM

conditions:
- Increased energy expenditure ("hypermetabolism")
- Oral-motor dysfunction
- Esophageal and gastric dysmotility
- Compromised intestinal function (maldigestion, malabsorption)
- Neurologic impairment
As shown in Item C1, enteral feedings are employed in a wide variety of clinical disease states to
achieve targeted nutrient intake. For conditions in which oral feedings cannot maintain nutritional
adequacy, enteral alimentation should be considered as either supportive or primary therapy for patients
retaining either partial or complete gastrointestinal function.
SUGGESTED READING:
Glassman MS, Woolf PK, Schwarz SM. Nutritional considerations in children with congenital heart
disease. In: Baker SB, Baker RD, Davis A, eds. Pediatric Enteral Nutrition. New York, NY: Chapman &
Hall; 1994: 340-350
Kleinman RE. Cardiac disease. In: Pediatric Nutrition Handbook. 6th ed. Elk Grove Village, IL:
American Academy of Pediatrics; 2009:981-1000
Leitch CA. Growth, nutrition and energy expenditure in pediatric heart failure. Progr Pediatr Cardiol.
2000;11:195-202. DOI: 10.1016/S1058-9813(00)00050-3. Abstract accessed December 2010 at:
http://www.ncbi.nlm.nih.gov/pubmed/10978712
Schwarz SM, Gewitz MH, See CC, et al. Enteral nutrition in infants with congenital heart disease and
growth failure. Pediatrics. 1990;86:368-373. Accessed December 2010 at:
http://pediatrics.aappublications.org/cgi/reprint/86/3/368
Schwarz SM. Feeding disorders in children with developmental disabilities. Infants & Young Children.
2003;16:317-330. Abstract accessed December 2010 at:
http://journals.lww.com/iycjournal/Abstract/2003/10000/Feeding_Disorders_in_Children_With_Developm
ental.5.aspx
Serrano M-S, Mannick EM. Consultation with the specialist: enteral nutrition. Pedatr Rev.
2003;24:417-423. DOI: 10.1542/pir.24-12-417. Accessed December 2010 at:
http://pedsinreview.aappublications.org/cgi/content/full/24/12/417

Copyright 2012 American Academy of Pediatrics

eh

Dr

_F

aq

eh

Dr

American Academy of Pediatrics

_F

aq

2012 PREP SA on CD-ROM

Question: 10
A 14-year-old boy suffers a nondisplaced fracture of his left radius and ulna while playing soccer. He
had a similar injury to his right radius and ulna 9 months ago. Physical examination reveals Sexual
Maturity Rating 2 pubic hair and testicular volume of 6 mL. A thorough review of his dietary habits
suggests that his daily intake of calcium and phosphorus are 800 mg each. He takes a 400 IU of vitamin
D supplement daily. Serum calcium measures 7.9 mg/dL (1.97 mmol/L), serum phosphorus measures
2.7 mg/dL (0.87 mmol/L), and 25-hydroxyvitamin D (25-OHD) measures 55 pg/mL (normal, 30 to 80
pg/mL).
Of the following, the MOST appropriate recommendation for this boy is to increase his

A. calcium and phosphorus intake to 1,300 mg/day


B. calcium and phosphorus intake to 2,000 mg/day
C. calcium intake to 1,000 mg/day
D. phosphorus intake to 1,000 mg/day
E. vitamin D supplementation to 2,000 IU/day

Copyright 2012 American Academy of Pediatrics

eh

Dr

_F

American Academy of Pediatrics

2012 PREP SA on CD-ROM

Critique: 10

Preferred Response: A

aq

The boy described in the vignette reportedly takes 800 mg of calcium, 800 mg of phosphorus, and
400 IU of vitamin D each day. The American Academy of Pediatrics recommends that preadolescents
and adolescents (9 to 18 years of age) consume 1,300 mg of both calcium and phosphorus daily.
Increasing the boy's calcium and phosphorus intake to 2,000 mg/day is not appropriate because excess
supplementation increases the risk of hypercalcemia, hypercalciuria, hyperphosphaturia, and renal
stones.
Because this boy has a normal vitamin D concentration, he should continue to supplement his diet
with the current recommended amount of 400 IU/day. Increasing supplementation to 2,000 IU/day is not
currently indicated, although this dose is unlikely to cause hypervitaminosis. Fortunately, vitamin D has a
very wide therapeutic window, and many physicians use high-dose (500,000 IU) or "stoss therapy"
(derived from the German "to push"), followed by maintenance with the recommended dietary allowance
(RDA) when treating vitamin D deficiency. Recent evidence of the high prevalence of vitamin D
deficiency in United States children supports the proposal for a higher vitamin D RDA.
Most children in the United States do not achieve the recommended daily allowance of calcium.
Therefore, a child's intake should be estimated at each health supervision visit. As much as 40% of total
lifetime bone mineral content is accrued during adolescence, with peak calcium accretion rates in the
bones occurring at an average of 12.5 years for girls and 14 years for boys. Thus, optimizing calcium
intake is particularly important during adolescence, and those who experience delayed puberty have an
increased risk for osteoporosis and fracture. Pediatricians should actively promote bone heath by
ensuring that their patients consume adequate daily calcium, phosphorus, and vitamin D and
encouraging them to participate in physical activity and weight-bearing exercises.
SUGGESTED READING:
Cashman KD, Flynn A. Optimal nutrition: calcium, magnesium and phosphorus. Proc Nutr Soc.
1999;58:477-487. DOI: 10.1017/S0029665199000622. Accessed April 2011 at:
http://journals.cambridge.org/action/displayFulltext?type=6&fid=795956&jid=PNS&volumeId=58&issueId
=02&aid=795952&bodyId=&membershipNumber=&societyETOCSession=&fulltextType=MR&fileId=S00
29665199000622
Greer FR, Krebs NF, Committee on Nutrition. Optimizing bone health and calcium intakes of infants,
children, and adolescents. Pediatrics. 2006;117:578-585. DOI: 10.1542/peds.2005-2822. Accessed
December 2010 at: www.pediatrics.org/cgi/doi/10.1542/peds.2005-2822
Wagner CL, Freer FR; Section on Breastfeeding and Committee on Nutrition. Prevention of rickets
and vitamin D deficiency in infants, children, and adolescents. Pediatrics. 2008;122:1142-1152. DOI:
10.1542/peds.2008-1862. Accessed December 2010 at: www.pediatrics.org/cgi/doi/10.1542/peds.2008-

Copyright 2012 American Academy of Pediatrics

eh

Dr

American Academy of Pediatrics

_F

aq

2012 PREP SA on CD-ROM

1862

Copyright 2012 American Academy of Pediatrics

eh

Dr

American Academy of Pediatrics

_F

aq

2012 PREP SA on CD-ROM

Question: 15
A mother brings in her 12-month-old infant for evaluation. One month ago she transitioned him from
human milk to a cow milk formula but noted that each time her son drank the formula, he developed a
rash around his mouth and scattered hives on his trunk. After speaking with her pediatrician she
switched to rice milk, which the infant has tolerated well. She reports that last week when her son
accidently drank cow milk, he developed a perioral rash and vomited once.
Of the following, you are MOST likely to counsel the mother that

A. bloody stools is a common presenting symptom


B. he is unlikely to tolerate a soy-based formula
C. he may tolerate foods containing cow milk if they are extensively heated
D. he will most likely develop other food allergies
E. negative allergy skin testing will exclude the diagnosis

Copyright 2012 American Academy of Pediatrics

eh

Dr

_F

American Academy of Pediatrics

2012 PREP SA on CD-ROM

Critique: 15

Preferred Response: C

aq

When evaluating a patient for an adverse food reaction, a careful history is important to distinguish
whether a reaction is immunoglobulin (Ig)E-mediated or non-IgE-mediated. IgE-mediated food
hypersensitivity, as described for the infant in the vignette, typically involves flushing, urticaria, rash, or
angioedema. Other IgE-mediated symptoms may include vomiting, bronchospasm, abdominal cramping,
and anaphylactic shock. Most patients are educated to avoid particular foods that cause adverse
reactions. However, recent studies have demonstrated that certain foods that contain heat-labile
proteins (eg, cow milk and eggs) are tolerated in most children if they are extensively heated. In a
recent study of 100 children ages 2 to 17 years, who were allergic to milk, 75% tolerated extensively
heated (EH) muffins and waffles that contained 1.3 g of milk protein. All children who tolerated EH food
challenges continued to eat one to three servings per day without reaction. Three months after
beginning the study, the children who tolerated the challenges demonstrated decreased skin test wheal
size and increased serum IgG4, both of which are encouraging immunologic trends toward possible
clinical resolution of their allergic reactions to unheated food.
Fortunately, although cow milk allergy is one of the most common IgE-mediated food allergies in
children, more than 90% of infants older than 6 months of age can safely be switched to a soy formula.
The risk for developing other food allergies is increased in children who have milk allergy, but only about
one third of patients develop other food allergies. After a thorough history and physical examination, IgE
skin prick testing or specific serum IgE testing should be performed. Interpretation of test results by an
allergist is recommended because the predictive value for each test result is different for different foods,
although a negative skin prick test result has a negative predictive value of 95% or greater for the most
common foods allergens (eg, milk, egg, wheat, soy, peanut, fish). Children at low risk for an IgEmediated food allergy, based on an unconvincing clinical history or a negative skin prick test, should
undergo a physician-supervised food challenge.
Non-IgE-mediated cow milk reactions include hematochezia (food protein colitis), abdominal pain,
and failure to thrive. Food protein colitis typically presents in the first 6 months after birth with bloodstreaked stools. Unlike IgE-mediated cow milk allergy, many infants who have food protein colitis or
enterocolitis continue to have symptoms with soy formula and typically require a change to a
hypoallergenic or amino-acid based formula.
SUGGESTED READING:
NAID-sponsored Expert Panel. Guidelines for the diagnosis and management of food allergy in the
united states: report of the NIAID-Sponsored Expert Panel. J Allergy Clin Immunol. 2010;126(suppl):S1S58. DOI: 10.1016/j.jaci.2010.10.007. Accessed January 2011 at:
http://www.jacionline.org/article/S0091-6749(10)01566-6/fulltext
Nowak-Wegrzyn A, Bloom KA, Sicherer SH, et al. Tolerance to extensively heated milk in children
with cow's milk allergy. J Allergy Clin Immunol. 2008;122:342-347. DOI: 10.1016/j.jaci.2008.05.043.
Accessed

Copyright 2012 American Academy of Pediatrics

eh

Dr

American Academy of Pediatrics

_F

aq

2012 PREP SA on CD-ROM

January 2011 at: http://www.jacionline.org/article/S0091-6749(08)01111-1/fulltext

Copyright 2012 American Academy of Pediatrics

eh

10

Dr

American Academy of Pediatrics

_F

aq

2012 PREP SA on CD-ROM

Question: 17
You are caring for a 7-year-old boy who developed a mid-gut volvulus at age 3, necessitating
resection of most of his small bowel. Only 10 cm of ileum and a few centimeters of duodenum remain,
and the child is dependent on parenteral nutrition for most of his protein-energy intake. He recently
developed a cough and dyspnea on exertion. Physical examination demonstrates a gallop cardiac
rhythm and the presence of rales at the lung bases. Chest radiography shows cardiomegaly.
Of the following, the nutritional deficiency that is MOST likely responsible for this boy's symptoms
and signs is

A. chromium
B. copper
C. manganese
D. selenium
E. zinc

Copyright 2012 American Academy of Pediatrics

eh

11

Dr

_F

American Academy of Pediatrics

2012 PREP SA on CD-ROM

Critique: 17

Preferred Response: D

aq

The child described in the vignette suffered a catastrophic intestinal event that left him with little
functioning bowel and dependent on parenteral nutrition for survival. The subsequent development of
cardiorespiratory dysfunction should alert the clinician to possible complications of central line
placement, including both infection and catheter dislodgement. However, particularly after long-term
parenteral nutrition support (4 years for this boy), factors related to composition of the nutrient infusate
must also be considered. The nutritional cause of what appears to be an emerging cardiomyopathy is
most likely a deficiency of the trace mineral selenium.
Selenium has only recently been identified as an essential nutrient, and deficiency in children has
been limited to those receiving long-term parenteral nutrition without selenium supplementation. This
trace mineral appears to be involved in the body's antioxidant defense system, and it is incorporated into
cellular, serum, and membrane-bound glutathione peroxidases. Selenium deficiency, where described,
has been associated with skin and hair pigment loss, macrocytosis, and in severe cases,
cardiomyopathy.
Trace minerals constitute less than 0.01% of total body weight in healthy individuals, but these
moieties are integral to numerous enzyme systems, functioning either as activating cofactors or as key
components of metalloenzymes. Symptomatic trace mineral deficiencies, although uncommon, occur
most frequently during periods of rapid growth (eg, infancy, particularly in preterm infants), in chronic
malnourished states, and as a consequence of intestinal maldigestion/malabsorption. In order of their
importance in childhood, the trace elements include iron, zinc, copper, fluoride, iodine, selenium,
manganese, cobalt, molybdenum, nickel, silicon, and vanadium. Calcium, phosphorus, and magnesium
together comprise 98% of the total body mineral content by weight (with the highest concentrations in
bone) and, therefore, are not considered trace elements. Recommended daily allowances (RDAs) for
zinc, copper, manganese, chromium, iodine, molybdenum, and selenium have been established by the
Food and Nutrition Board of the Institute of Medicine.
As noted, trace mineral deficiencies are uncommon in infants and children, occurring most frequently
in the setting of severe malnutrition, often associated with chronic diarrheal states (prominent in the
developing world) or as a consequence of disorders manifested by intestinal malabsorption/maldigestion
(eg, cystic fibrosis, short bowel syndrome, inflammatory bowel disease). For patients requiring long-term
parenteral nutrition support, current formulations of nutrient solutions are supplemented with trace
minerals, making deficiency states much less prevalent than in the past.
Zinc deficiency is the most often reported trace mineral deficiency after iron. Features of abnormal
zinc nutriture include acro-orificial skin lesions, diarrhea, increased susceptibility to infection, and growth
retardation. These clinical findings characterize acrodermatitis enteropathica, an autosomal recessive
disorder of zinc metabolism resulting from a mutation in the gene for intestinal brush-border membrane
ZIP4, a transporter that regulates enterocyte zinc uptake. Affected patients manifest severe deficiency.
Less marked zinc deficiency states may be difficult to recognize but should be suspected in at-risk
individuals who demonstrate slow growth, compromised immunity with frequent infections, and anorexia
due to loss of taste sensation (hypogeusia).

Copyright 2012 American Academy of Pediatrics

eh

12

Dr

American Academy of Pediatrics

_F

aq

2012 PREP SA on CD-ROM

The greatest risk for copper deficiency is seen in preterm infants in whom hepatic copper stores are
low because this trace mineral accumulates in the liver during the third trimester of pregnancy. However,
in this and in other at-risk clinical situations (rapid growth rate, malabsorption syndromes, severe
malnutrition), the deficiency state is generally not seen unless both endogenous copper stores and
dietary copper intake are low. Signs of copper deficiency include neutropenia, hypochromic anemia
unresponsive to iron administration, bone abnormalities, and hair and skin depigmentation. Menkes
syndrome is an X-linked recessive disorder of copper metabolism caused by a defect in the protein
ATP7A, resulting in an inability to export intracellular copper. This syndrome is characterized by anemia,
steely hair, and progressive cerebral degeneration and has a poor prognosis despite aggressive copper
supplementation.
Other trace mineral deficiencies have generally been reported only as a consequence of severe
malnutrition. Chromium functions as a cofactor for insulin, and the deficiency state, although rare, may
be characterized by impaired glucose, fat, and protein metabolism and by growth retardation.
Manganese is a required cofactor for some enzymes, including arginase and pyruvate carboxylase. Its
nutrient status is difficult to assess because manganese is present in very low concentrations in the
bloodstream. The essential nature of this trace mineral in the human diet has not been demonstrated,
and the dietary requirement for manganese is very small. To date only one possible case of manganese
deficiency has been described.
SUGGESTED READING:
Kleinman RE. Trace elements. In: Pediatric Nutrition Handbook. 6th ed. Elk Grove Village, IL:
American Academy of Pediatrics;2009:423-452
Krebs NF, Westcott JE, Arnold TD, et al. Abnormalities in zinc homeostasis in young infants with
cystic fibrosis. Pediatr Res. 2000;48:256-261. Abstract accessed December 2010 at:
http://www.ncbi.nlm.nih.gov/pubmed/10926304
Litov RE, Combs GF Jr. Selenium in pediatric nutrition. Pediatrics. 1991;87:339-351. Accessed
December 2010 at: http://pediatrics.aappublications.org/cgi/reprint/87/3/339
Lockitch G, Taylor GP, Wong LT, et al. Cardiomyopathy associated with nonendemic selenium
deficiency in a Caucasian adolescent. Am J Clin Nutr. 1990;52:572-577. Accessed December 2010 at:
http://www.ajcn.org/content/52/3/572.long
Mertz W. Chromium in human nutrition: a review. J Nutr. 1993;123:626-633. Accessed December
2010 at: http://jn.nutrition.org/content/123/4/626.long
Olivares M, Araya M, Uauy R. Copper homeostasis in infant nutrition: deficit and excess. J Pediatr

Copyright 2012 American Academy of Pediatrics

eh

13

Dr

American Academy of Pediatrics

_F

aq

2012 PREP SA on CD-ROM

Gastroenterol Nutr 2000;31:102-111. Accessed December 2010 at:


http://journals.lww.com/jpgn/Fulltext/2000/08000/Copper Homeostasis_in
infant_Nutrition__Deficit.4.aspx

Copyright 2012 American Academy of Pediatrics

eh

14

Dr

American Academy of Pediatrics

_F

aq

2012 PREP SA on CD-ROM

Question: 35
A 5-month-old male infant presents to your office because of pallor and irritability. He was born at
term following an uncomplicated pregnancy and delivery and had a birthweight of 3,150 g. Because of
newborn screening results, a sweat chloride examination was performed at 1 month of age that
confirmed a diagnosis of cystic fibrosis. Since birth, the baby has been exclusively breastfed. At the time
of diagnosis, pancreatic enzyme supplementation was begun and now includes 8,000 units of lipase per
nursing session. Approximately 1 week ago, the mother noted that the baby was "breathing fast" and
appeared very pale. On physical examination today, the well-developed infant has a weight of 6.0 kg,
heart rate of 160 beats/min, and respiratory rate of 40 breaths/min. You also note conjunctival and
mucous membrane pallor and a liver edge palpable 2 cm below the right costal margin. Laboratory
results include:
- Hemoglobin, 9.0 g/dL (90 g/L)
- White blood cell count, 9.0x103/mcL (9.0x109/L)
- Albumin, 3.8 g/dL (38 g/L)
- Reticulocyte count, 12.5% (0.125)
A blood smear demonstrates polychromasia and numerous schistocytes (Item Q35).
Of the following, the MOST appropriate treatment for this infant is

A. alpha-tocopherol
B. ascorbic acid
C. cyanocobalamin
D. folic acid
E. thiamine

Copyright 2012 American Academy of Pediatrics

eh

15

Dr

_F

aq

eh

Dr

_F

American Academy of Pediatrics

2012 PREP SA on CD-ROM

Critique: 35

Preferred Response: A

aq

As per current recommendations of the Cystic Fibrosis (CF) Foundation, the infant described in the
vignette is breastfed and supplemented with exogenous pancreatic enzymes. The most common
nutritional problems in children who have CF are the consequences of fat and fat-soluble vitamin
malabsorption due to pancreatic insufficiency and often cholestasis. The infant described in the vignette
has developed a hemolytic anemia, which is most consistent with a deficiency in alpha-tocopherol
(vitamin E). Deficiencies of the water-soluble vitamins (including ascorbic acid, folic acid,
cyanocobalamin, and thiamine) are uncommon and would be the result of severe malnutrition in
association with reduced dietary intake of these vitamins.
Achievement of normal growth is a critically important objective in the care of patients who have CF.
This goal requires appropriate energy and nutrient intake that must account for disease-related
disturbances in gastrointestinal and pulmonary function. Studies have shown that poor clinical outcomes
are associated with undernutrition in patients who have CF. As stated in the 2002 Consensus Report,
"The CF care team should monitor growth, provide anticipatory counseling, and plan intervention
strategies when patients are at risk for undernutrition or are diagnosed with nutritional failure. A
registered dietitian must be a part of the team to provide the discipline-specific expertise needed for
optimal nutritional management and may, along with other team members such as the physician, nurse,
social worker or a psychologist, provide expertise concerning developmental and behavioral aspects of
eating. Prevention and early intervention are most successful in combating nutritional failure."
Specific nutritional deficits in CF fall into two broad categories: overall protein-energy malnutrition due
to insufficient macronutrient intake and specific nutritional deficiencies related to pancreatic insufficiency
and impaired digestion/absorption. Guidelines have been established for pancreatic enzyme
supplementation for infants (Item C35A) and young children using both standard and enteric-coated
preparations. Similar guidelines for enzyme dosing have been established for older children and adults.
The goal is to achieve normal growth and weight gain by appropriate dietary guidance but also to limit
enzyme supplements to less than 2,500 U lipase/kg per meal to avoid fibrosing colonopathy, a
complication of high-dose enzyme supplementation. With the possible exception of patients who have
severe lung disease, the use of oral protein-energy supplements is controversial. A recent systematic
study suggests that dietary advice alone is a satisfactory approach for patients who have CF and
moderate malnutrition and that specialized supplemental formulas are of little value.
Careful monitoring of patients who have CF is required not only to assure sufficient energy intake and
optimize pancreatic enzyme dosing, but also to assure adequate supplementation with the fat-soluble
vitamins A, D, E, and K. At diagnosis, most infants require 125% of the recommended dietary allowance
for energy until catch-up growth is achieved and a long-term nutritional plan is implemented.
Malabsorption of fat-soluble vitamins is a particular concern in those who have pancreatic insufficiency;
in one study, biochemical evidence of fat-soluble vitamin deficiency was found in almost 50% of patients
studied. Accordingly, the required intake of these vitamins for those who have CF may range from 2 to
20 times the daily recommended intake (with the exception of vitamin K, where required intake values
are unknown), as shown in (Item C35B). These vitamins should be provided as their water-soluble

Copyright 2012 American Academy of Pediatrics

eh

17

Dr

American Academy of Pediatrics

_F

aq

2012 PREP SA on CD-ROM

assure maximum absorption. A number of formulations specifically developed for CF patients are
available. Compliance with vitamin therapy is an important objective in overall CF care, and
concentrations of vitamins A, E, and D should be measured annually. In CF patients, adverse effects of
fat-soluble vitamin deficiencies are not uncommon, and they include:
- Vitamin A: xerophthalmia
- Vitamin E: neuropathy, retinopathy, and hemolytic anemia
- Vitamin D: osteopenia, fractures
- Vitamin K: clotting abnormalities
The risk for these deficiency states is heightened in patients who manifest cholestasis in addition to
pancreatic insufficiency.
SUGGESTED READING:
Borowitz D, Baker RD, Stallings V. Consensus report on nutrition for pediatric patients with cystic
fibrosis. J Pediatr Gastroenterol Nutr 2002;35:246-259. Accessed December 2010 at:
http://journals.lww.com/jpgn/Fulltext/2002/09000/Consensus_Report_on_Nutrition_for_Pediatric.4.aspx
Feranchak AP, Sontag MK, Wagener JS, Hammond KB, Accurso FJ, Sokol RJ. Prospective, longterm study of fat-soluble vitamin status in children with cystic fibrosis identified by newborn screen. J
Pediatr. 1999;135:601-610. Abstract accessed December 2010 at:
http://www.ncbi.nlm.nih.gov/pubmed/10547249
Kleinman RE. Nutrition in cystic fibrosis. In: Pediatric Nutrition Handbook. 6th ed. Elk Grove Village,
IL: American Academy of Pediatrics; 2009:1001-1020
Rayner RJ. Fat-soluble vitamins in cystic fibrosis. Proceedings of the Nutrition Society. 1992;51:245250. DOI: 10.1079/PNS19920035. Accessed December 2010 at:
http://journals.cambridge.org/action/displayFulltext?type=1&fid=752416&jid=PNS&volumeId=51&issueId
=02&aid=752408&bodyId=&membershipNumber=&societyETOCSession=
Smyth RL, Walters S. Oral calorie supplements for cystic fibrosis. Cochrane Database Syst Rev.
2007;1: CD000406. DOI: 10.1002/14651858.CD000406.pub2. Accessed December 2010 at:
http://onlinelibrary.wiley.com/o/cochrane/clsysrev/articles/CD000406/frame.html
Stallings VA, Stark LJ, Robinson KA, Feranchak AP, Quinton H; Clinical Practice Guidelines on
Growth and Nutrition Subcommittee, Ad Hoc Working Group. Evidence-based practice
recommendations for nutrition-related management of children and adults with cystic fibrosis and
pancreatic insufficiency: results of a systematic review. J Am Diet Assoc. 2008;108:832-839. DOI:

Copyright 2012 American Academy of Pediatrics

eh

18

Dr

American Academy of Pediatrics

_F

aq

2012 PREP SA on CD-ROM

Abstract accessed December 2010 at: http://www.ncbi.nlm.nih.gov/pubmed/18442507

Copyright 2012 American Academy of Pediatrics

eh

19

Dr

_F

aq

eh

Dr

_F

aq

eh

Dr

American Academy of Pediatrics

_F

aq

2012 PREP SA on CD-ROM

Question: 38
A 14-year-old girl presents for an office visit because of fatigue and increased sleepiness. Menarche
occurred when she was 11 years old, but her menses have always been irregular, and she has noted
some "spotting" at times between menses. She also reports a normal diet with occasional constipation.
Physical examination reveals slightly pale conjunctivae and oral mucosa. Her heart rate is 110
beats/min, she has a grade I/VI systolic ejection murmur heard best at the left sternal border, and her
blood pressure measures 110/70 mm Hg supine and 105/60 mm Hg sitting. Abdominal examination is
negative for tenderness or mass. There is a small amount of blood on a sanitary pad on external
genitourinary examination
Laboratory studies reveal:
- White blood cell count, 14.0x103/mcL (14.0x109/L)
- Hemoglobin, 9.9 mg/dL (99 g/L)
- Hematocrit, 29% (0.29)
- Mean corpuscular volume, 65 fL
- Reticulocyte count, 0.2% (0.002)
Of the following, the BEST treatment for this patient is

A. daily multivitamin with iron


B. dietary counseling for iron-rich foods
C. intramuscular iron administration
D. oral supplementation with folic acid
E. prescription for oral ferrous sulfate three times daily

Copyright 2012 American Academy of Pediatrics

eh

22

Dr

_F

American Academy of Pediatrics

2012 PREP SA on CD-ROM

Critique: 38

Preferred Response: E

aq

The girl described in the vignette has both historical and physical examination findings consistent with
a diagnosis of iron deficiency anemia. Typical laboratory findings in patients who have iron deficiency
include a low mean corpuscular volume and mean corpuscular hemoglobin; a hypochromic, microcytic
peripheral blood smear; and a normal or low reticulocyte count. Iron deficiency is common in adolescent
girls and most typically results from menstrual blood loss and inadequate dietary iron intake. However,
the clinician must be vigilant for more insidious causes, including gastrointestinal blood loss due to
inflammatory bowel disease, eating disorders, autoimmune diseases such as lupus erythematosus, and
abdominal trauma.
The most likely cause of the anemia for the girl described in the vignette is menstrual blood loss or
dietary iron insufficiency. Appropriate initial management includes evaluation and management of
menorrhagia, perhaps including oral hormone therapy to diminish dysfunctional uterine bleeding, and
oral iron supplementation with ferrous sulfate three times daily. The amount of iron in a multivitamin is
insufficient to treat iron deficiency; it is intended only to prevent iron deficiency.
The typical American teenager's diet is low in iron-rich foods, and recent reports suggest that
overweight children may be at special risk for iron deficiency. However, counseling about iron-rich foods
is not the best treatment for the adolescent described in the vignette because oral ferrous sulfate is
needed to reverse the anemia rapidly.
Folic acid supplementation is unlikely to reverse the anemia because the laboratory findings are not
suggestive of an anemia due to folic acid deficiency (macrocytosis). Intramuscular iron administration is
not indicated for this adolescent because these injections are painful, may lead to permanent skin
discoloration, and have been associated with anaphylactic reactions. In addition, intramuscular iron is
neither more efficacious nor rapidly acting than oral iron.
SUGGESTED READING:
Baker RD, Greer FR, The Committee on Nutrition. Diagnosis and prevention of iron deficiency and
iron-deficiency anemia in infants and young children (0-3 years of age). Pediatrics. 2010;126:1040-1050.
DOI: 10.1542/peds.2010-2576. Accessed January 2011 at:
http://pediatrics.aappublications.org/cgi/content/full/126/5/1040
Lerner NB, Sills R. Anemias of inadequate production: iron deficiency anemia. In: Kliegman RM,
Stanton BF, St. Geme JW III, Schor NF, and Behrman RE, eds. Nelson Textbook of Pediatrics. 19th ed.
Philadelphia, PA: Saunders Elsevier; 2011:1655-1658
Janus J, Moerschel SK. Evaluation of anemia in children. Am Fam Physician. 2010;81:1462-1471.
Abstract accessed January 2011 at: http://www.ncbi.nlm.nih.gov/pubmed/20540485
Killip S, Bennett JM, Chambers MD. Iron deficiency anemia. Am Fam Physician. 2007;75:671-678.

Copyright 2012 American Academy of Pediatrics

eh

23

Dr

American Academy of Pediatrics

_F

aq

2012 PREP SA on CD-ROM

Accessed January 2011 at: http://www.aafp.org/afp/2007/0301/p671.html


Nead KG, Halterman JS, Kaczorowski JM, Auinger P, Weitzman M. Overweight children and
adolescents: a risk group for iron deficiency. Pediatrics. 2004;114:104-108. Accessed January 2011 at:
http://pediatrics.aappublications.org/cgi/content/full/114/1/104

Copyright 2012 American Academy of Pediatrics

eh

24

Dr

American Academy of Pediatrics

_F

aq

2012 PREP SA on CD-ROM

Question: 229
You are evaluating a 28 weeks gestational age infant who was recently discharged from the neonatal
intensive care unit after a 4-month hospital stay. She was delivered prematurely due to severe
pregnancy-induced hypertension and intrauterine growth restriction, with a birthweight of 700 g (3rd
percentile). She had a complicated hospital course that included severe respiratory distress syndrome
and necrotizing enterocolitis, with prolonged periods of parenteral nutrition and poor weight gain. She
now is breathing room air and receiving oral feeding of 150 mL/kg per day of human milk fortified to 24
kcal/oz. Her weight in your office is 2,500 g (<3rd percentile). Her parents have many concerns about
her small size and neurodevelopmental outcome.
Of the following, the MOST appropriate counseling for this family is that

A. early growth deficits are not associated with poor neurodevelopmental outcomes
B. feeding strategies that provide 90 kcal/kg per day maintain optimal growth
C. higher protein intake after birth improves neurodevelopmental indices at 18 months
D. increased amounts of lipid after hospital discharge support brain development
E. nutrition in the second year after birth is critical to neurodevelopmental outcome

Copyright 2012 American Academy of Pediatrics

eh

25

Dr

_F

American Academy of Pediatrics

2012 PREP SA on CD-ROM

Critique: 229

Preferred Response: C

aq

Preterm infants are at increased risk of poor growth and decreased neurodevelopmental indices due
to inadequate nutrition after birth, but higher protein intakes are believed to improve these outcome
measures. Studies have demonstrated that higher protein intakes in parenteral and enteral nutrition are
associated with increased protein accretion and weight gain as well as improved neurodevelopmental
outcomes at both 18 months and 7 to 8 years of age. Early initiation of parenteral amino acids at 3 g/kg
per day or more by the fifth postnatal day in very low-birthweight infants has been associated with better
growth at 36 weeks postmenstrual age and less suboptimal head growth at 18 months corrected
gestational age. Current guidelines for very low-birthweight infants recommend a protein content of 3.6
g/kg per day at an average energy intake of 120 kcal/kg per day. Evidence of inadequate protein intake,
such as a low serum urea nitrogen concentration (<4 mg/dL[1.4 mmol/L]), may suggest the need to
increase protein supplementation.
Poor growth in the preterm infant is also linked to daily energy intake. Improved neurodevelopment
and growth are associated with weight gain of more than 18 g/kg per day and head circumference
growth of more than 0.9 cm/wk during hospitalization after birth. The estimated daily energy requirement
of a preterm infant for maintenance and growth is 120 kcal/kg per day. The actual energy requirement
varies from infant to infant, with conditions such as intrauterine growth restriction, chronic lung disease,
congestive heart failure, short bowel syndrome, and infection often increasing the energy needs.
Increases in feeding volumes or caloric content may be needed to sustain a weight gain of more than 18
g/kg per day.
The first year after birth is critical for nutritional compensation to allow catch-up in both somatic and
brain growth; the impact is limited after this time period. Continued attention to energy and protein
content in the diet of preterm infants after discharge from the hospital is essential. No data support
increasing the amount of lipid after discharge for enhanced brain development.
SUGGESTED READING:
Adamkin DH. Nutrition management of the very low-birthweight infant. II. Optimizing enteral nutrition
and postdischarge nutrition. NeoReviews. 2006;7:e608-e613. DOI: 10.1542/neo.7-12-e608. Accessed
January 2011 at: http://neoreviews.aappublications.org/cgi/content/full/7/12/e608
American Academy of Pediatrics Committee on Nutrition. Nutritional needs of the preterm infant. In:
Kleinman RE, ed. Pediatric Nutrition Handbook. 6th ed. Elk Grove Village, IL: American Academy of
Pediatrics; 2009:79-112
Franz AR, Pohlandt F, Bode H, et al. Intrauterine, early neonatal, and postdischarge growth and
neurodevelopmental outcome at 5.4 years in extremely preterm infants after intensive neonatal
nutritional support. Pediatrics. 2009;123:e101-e109. DOI: 10.1542/peds.2008-1352. Accessed January
2011 at: http://pediatrics.aappublications.

Copyright 2012 American Academy of Pediatrics

eh

26

Dr

American Academy of Pediatrics

_F

aq

2012 PREP SA on CD-ROM

org/cgi/content/full/123/1/e101
Stephens BE, Walden RV, Gargus RA, et al. First-week protein and energy intakes are associated
with 18-month developmental outcomes in extremely low birth weight infants. Pediatrics. 2009;123:13371343. DOI: 10.1542/peds.2008-0211. Accessed January 2011 at:
http://pediatrics.aappublications.org/cgi/content/full/123/5/1337

Copyright 2012 American Academy of Pediatrics

eh

27

Dr

_F

aq

eh

2011 PREP SA on CD-ROM


Question: 17

A 6-month-old infant presented in the newborn period with intestinal malrotation and mid-gut
volvulus. Emergency surgery resulted in resection of his entire small bowel, except for 7 cm of
duodenum. He has been maintained on parenteral nutrition since then and is listed for small bowel
transplantation at the regional transplant center. His parenteral nutrition regimen provides 120
kcal/kg per day and includes 20% dextrose, 3 g/120 kcal per day amino acids, and 3 g/kg per day
lipids. He has had increasing jaundice over the past month. On physical examination, the alert,
afebrile, and icteric infant has a firm liver edge palpable 3.5 cm below the right costal margin.
Laboratory data include:
Total bilirubin, 12.5 mg/dL (213.8 mcmol/L)
Direct bilirubin, 8.0 mg/dL (136.8 mcmol/L)
Alanine aminotransferase, 200 units/L
Aspartate aminotransferase, 150 units/L
Gamma-glutamyl transpeptidase, 180 units/L
Of the following, the MOST appropriate treatment is to

A. add phenobarbital 5 mg/kg per day orally


B. add ursodeoxycholic acid 20 mg/kg per day orally
C. decrease amino acids to 1.5 g/120 kcal per day
D. decrease dextrose to 10%
E. decrease lipids to 1.0 g/kg per day

Copyright 2010 by the American Academy of Pediatrics

page 1

Dr

_F

aq

eh

2011 PREP SA on CD-ROM


Critique: 17

Preferred Response: E

Parenteral nutrition (PN) is a life-sustaining therapy for patients whose limited


gastrointestinal function does not permit adequate enteral absorption of nutrients. Despite its
ability to maintain nutritional status and support growth, long-term PN has been associated with
numerous serious and sometimes life-threatening complications, including bloodstream infections
due to bacterial and fungal contamination of central venous catheters, metabolic derangements,
and PN-associated liver disease (PNALD). The infant described in the vignette has PNALD, an
extremely challenging problem that is especially common in patients who have short-bowel
syndrome, for reasons that are not completely understood. Although the causes of
hepatocellular dysfunction and cholestasis (indicated by direct hyperbilirubinemia and elevated
gamma-glutamyl transpeptidase values) in this clinical setting may be multifactorial, current data
suggest that excess intravenous lipids or specific elements in lipid emulsions play a major role in
the pathogenesis of PNALD. Accordingly, the most reasonable approach for the infant in the
vignette is to decrease the amount of infused lipid.
For the patient who has an extremely short bowel, PN serves as a "bridge therapy."
Because the infant in the vignette retains only a short length of duodenum, the prognosis is dire
for achieving any significant small bowel adaptation that can lead to recovery of intestinal
absorptive function. Therefore, long-term survival clearly depends upon a successful small
intestinal transplant. For patients in whom residual bowel length is sufficient to undergo adaptive
change and support some level of fluid and nutrient absorption, early reintroduction of enteral
feedings remains the best option for limiting the occurrence and severity of PNALD.
Careful ongoing clinical assessment is essential for all patients receiving PN to identify and
treat potential complications of this therapy. In addition to routine biochemical monitoring (Item
C17), scrupulous attention must be paid to strict adherence to aseptic technique, both in
parenteral fluid preparation and central line care. Ideally, central venous catheter access,
including care required for routine dressing changes, should be carried out only by personnel
trained and credentialed in line management. Such precautions should limit PN-associated
complications, although PNALD remains a vexing and often unavoidable problem.
All major nutrients infused in PN solutions, as well as other factors (eg, infection, hepatic
accumulation of bile acids) have been implicated in the pathogenesis of PNALD. However,
advances in the composition and delivery of nutrients to infants receiving PN have resulted in
amino acid profiles that mimic those during breastfeeding, and control of carbohydrate intake
avoids hyperglycemia and wide swings in blood glucose values. Recently, increased focus has
been placed on the role of intravenous lipids in the pathogenesis of PNALD. Both excess lipid
administration and phytosterols, present in soybean emulsions used for PN, have been
postulated as causative factors. Reducing infused lipids to 1.0 g/kg per day in this infant should
ameliorate PNALD at least partially while preventing the onset of essential fatty acid deficiency.
One recent study in two infants reported dramatic improvement in PNALD following replacement
of the standard omega-6 fatty acid-based lipid infusate with a phytosterol-free omega-3 fatty
acid emulsion.
Control of carbohydrate and protein intake, as demonstrated in the vignette, should limit PNassociated metabolic derangements. Accordingly, these major nutrients are much less likely to be
implicated in the pathogenesis of PNALD in this infant, and reduction in parenteral glucose or

Copyright 2010 by the American Academy of Pediatrics

page 2

Dr

_F

aq

eh

2011 PREP SA on CD-ROM

amino acid intake would not be expected to ameliorate cholestasis. Early studies suggested that
phenobarbital enhances bile salt-independent biliary flow. However, its efficacy in this condition
has not been demonstrated, and this therapy is not recommended for use in any pediatric
cholestatic state. Ursodeoxycholic acid is a bile acid that does not form micelles, undergoes
enterohepatic circulation, and increases hepatocellular bile excretion. However, the extreme
short bowel with absent ileum (the site of active bile salt reabsorption) described for the infant in
the vignette precludes its effectiveness.
Suggested reading:
Clayton PT, Whitfield P, Iyer K. The role of phytosterols in the pathogenesis of liver complications
of pediatric parenteral nutrition. Nutrition. 1998;14:158-164. Abstract available at:
http://www.ncbi.nlm.nih.gov/pubmed/9437703
Colomb V, Jobert-Giraud A, Lacaille F, Goulet O, Fournet JC, Ricour C. The role of lipid emulsions
in cholestasis associated with long-term parenteral nutrition in children. JPEN J Parenter Enteral
Nutr. 2000;24:345-350. Abstract available at: http://www.ncbi.nlm.nih.gov/pubmed/11071594
Gura KM, Duggan CP, Collier SB, et al. Reversal of parenteral nutritionassociated liver disease
in two infants with short bowel syndrome using parenteral fish oil: implications for future
management. Pediatrics. 2006;118:e197-e201. DOI: 10.1542/peds.2005-2662. Available at:
http://pediatrics.aappublications.org/cgi/content/full/118/1/e197
Kleinman RE. Parenteral nutrition. In: Pediatric Nutrition Handbook. 6th ed. Elk Grove Village, Ill:
American Academy of Pediatrics; 2009:519-540

Copyright 2010 by the American Academy of Pediatrics

page 3

Dr

_F

aq

eh

2011 PREP SA on CD-ROM


Critique: 17

Copyright 2010 by the American Academy of Pediatrics

page 4

Dr

_F

aq

eh

2011 PREP SA on CD-ROM


Question: 18

During the 1-week health supervision visit, a mother who is exclusively breastfeeding asks about
vitamin and iron supplementation for her healthy term infant. She explains that her previous child,
who was born at 30 weeks gestation, was discharged with an oral iron supplement and
vitamins.
Of the following, the MOST appropriate oral supplement to initiate for this infant at this visit is

A. calcium
B. folic acid
C. iron
D. vitamin D
E. vitamin K

Copyright 2010 by the American Academy of Pediatrics

page 5

Dr

_F

aq

eh

2011 PREP SA on CD-ROM


Critique: 18

Preferred Response: D

The most appropriate oral supplement to initiate at 1 week of age for an exclusively
breastfeeding term infant is vitamin D. In 2008, new guidelines from the American Academy of
Pediatrics recommended that breastfed and partially breastfed infants be supplemented with 400
IU of vitamin D daily within days of birth. This change in the recommended amount of vitamin D
supplementation arose because of continued reports of rickets in breastfed infants. Exclusive
breastfeeding without adequate sun exposure or vitamin D supplementation is a risk factor for
vitamin D deficiency and rickets.
Term newborns are assumed to have adequate iron stores for the first 4 to 6 months after
birth. The adequate intake of iron for the first 6 postnatal months is 0.27 mg/day and is estimated
from the content of iron in human milk and its high bioavailability. Recent studies have shown that
exclusively breastfed infants who received iron supplementation between 1 and 6 months of
age had improved hemoglobin concentrations at 6 months of age as well as better visual acuity
and higher Bayley Psychomotor Indices at 13 month of age when compared with
unsupplemented peers. The AAP Committee on Nutrition recently recommended that exclusively
breastfed term infants receive a supplement of elemental iron at 1 mg/kg per day, starting at 4
months of age and continuing until appropriate iron-containing foods have been introduced. The
preterm infant has lower iron content than the term infant and requires initiation of iron
supplementation between 2 and 4 weeks of age and extending through 12 months of age.
The term infant has adequate calcium stores, in contrast to the preterm infant, who is born
during the period when 80% of calcium, phosphorus, and magnesium are accrued. The term
infant also has adequate folate stores. The routine use of a standard dose of intramuscular
vitamin K at birth minimizes the risk of hemorrhagic disease of the newborn, which results from
vitamin K deficiency.
As a result of reviewing this information, do you intend to make a change in practice
to provide better patient care?
Yes
No
Suggested reading:
Baker RD, Greer FR, and the Committee on Nutrition. Diagnosis and prevention of iron deficiency
and iron-deficiency anemia in infants and young children (03 years of age). Pediatrics.
2010;126:1040-1050. Available at:
http://pediatrics.aappublications.org/cgi/content/abstract/peds.2010-2576v1
Dee DL, Sharma AJ, Cogswell ME, Grummer-Strawn LM, Fein SB, Scanlon KS. Sources of
supplemental iron among breastfed infants during the first year of life. Pediatrics.
2008;122(suppl 2):S98-S104. DOI: 10.1542/peds.2008-1315m. Available at:
http://pediatrics.aappublications.org/cgi/content/full/122/Supplement_2/S98
Kleinman RE. Iron. In: Pediatric Nutrition Handbook. 6th ed. Elk Grove Village, Ill: American
Academy of Pediatrics; 2009:403-422
Copyright 2010 by the American Academy of Pediatrics

page 6

Dr

_F

aq

eh

2011 PREP SA on CD-ROM

Wagner CL, Greer FR, and the Section on Breastfeeding and Committee on Nutrition. Prevention
of rickets and vitamin D deficiency in infants, children, and adolescents. Pediatrics.
2008;122:1142-1152. DOI: 10.1542/peds.2008-1862. Available at:
http://pediatrics.aappublications.org/cgi/content/full/122/5/1142

Copyright 2010 by the American Academy of Pediatrics

page 7

Dr

_F

aq

eh

2011 PREP SA on CD-ROM


Question: 34

A mother in your pediatric practice recently delivered a 28 weeks gestation infant who is in the
neonatal intensive care unit. The woman exclusively breastfed her previous child, who was born
at 36 weeks gestation. She is concerned that something is wrong with her milk for this infant
because it is being combined with human milk fortifier before being given to her infant. You
reassure her by explaining that fortification helps to meet the additional needs of her preterm
infant.
Of the following, the MOST important role of such fortification is to

A. augment the immunologic properties of human milk


B. boost the carbohydrate content of human milk
C. decrease the osmolality of human milk
D. enhance the absorption of iron from human milk
E. increase the protein content of human milk

Copyright 2010 by the American Academy of Pediatrics

page 8

Dr

_F

aq

eh

2011 PREP SA on CD-ROM


Critique: 34

Preferred Response: E

The protein content of human milk must be increased to meet the requirements of a preterm
infant. The estimated protein requirement for a preterm infant is 3.0 to 4.0 mg/kg per day
compared to 1.5 to 2.0 mg/kg per day for the term infant. The requirement for the preterm infant
does not include additional allowances for catch-up growth that are due to losses of lean body
weight prior to the infant surpassing birthweight. If these allowances are included, the
recommended protein content increases to 3.4 to 4.2 mg/kg per day.
Milk supplied by the preterm infants mother is the preferred enteral feeding. Although milk
produced by a mother who delivers prematurely may have increased protein content compared
to term milk, the protein content declines in the first weeks of lactation to that of term milk. Human
milk fortifier supplies additional protein, which has been shown to increase concentrations of
blood urea nitrogen and weight gain in the enterally feeding preterm infant.
Human milk fortifier does not augment the immunologic properties of human milk. Human milk
contains secretory immunoglobulin A as well as other factors such as lactoferrin and lysozyme
that confer immunologic protection upon the infant. The carbohydrate content of preterm human
milk may be slightly lower than term human milk, but it is well tolerated in spite of the intestinal
lactase functioning at 30% of the level of a term infant. The iron content of human milk is lower
than that of iron-fortified formula, but nearly 50% is absorbed. Human milk fortifier does not
significantly increase the carbohydrate content or enhance the absorption of iron. Of note, the
addition of human milk fortifier does increase the osmolality of enteral human milk feedings.
Suggested reading:
Adamkin DH. Nutrition management of the very-low birthweight infant: II. Optimizing enteral
nutrition and postdischarge nutrition. NeoReviews. 2006;7:e608-e614. Available at:
http://neoreviews.aappublications.org/cgi/content/full/7/12/e608
Kleinman RE. Nutritional needs of the preterm infant. In: Pediatric Nutrition Handbook. 6th ed. Elk
Grove Village, Ill: American Academy of Pediatrics; 2009:79-112
Kuschel CA, Harding JE. Multicomponent fortified human milk for promoting growth in preterm
infants. Cochrane Database Syst Rev. 2004;1:CD000343. DOI:
10.1002/14651858.CD000343.pub2. Available at:
http://www.mrw.interscience.wiley.com/cochrane/clsysrev/articles/CD000343/frame.html
Premji SS, Fenton TR, Sauve RS. Higher versus lower protein intake in formula-fed low birth
weight infants. Cochrane Database Syst Rev. 2006;1:CD003959.
DOI:10.1002/14651858.CD003959.pub2. Available at:
http://www.mrw.interscience.wiley.com/cochrane/clsysrev/articles/CD003959/frame.html

Copyright 2010 by the American Academy of Pediatrics

page 9

Dr

_F

aq

eh

2011 PREP SA on CD-ROM


Question: 145

You are caring for an 18-year-old girl who was diagnosed 6 months ago with Crohn disease
affecting the terminal ileum and colon. Her medications include prednisone 10 mg/day,
sulfasalazine 1 g three times per day, and 6-mercaptopurine 75 mg/day. She has experienced an
excellent symptomatic response to treatment and presents today complaining only of occasional
streaks of blood in an otherwise normal daily bowel movement. Since diagnosis, she has gained
2.5 kg on a regular diet with only iron supplementation.
Of the following, the girl is MOST at risk of developing a deficiency of

A. copper
B. folic acid
C. selenium
D. vitamin D
E. zinc

Copyright 2010 by the American Academy of Pediatrics

page 10

Dr

_F

aq

eh

2011 PREP SA on CD-ROM


Critique: 145

Preferred Response: B

Malnutrition is a common finding in inflammatory bowel disease (IBD), particularly Crohn


disease. Although the cause is multifactorial, decreased energy intake is the most frequent
contributing factor. Other causes include increased energy requirements, enteric losses,
malabsorption, and drug-nutrient interactions. As a result, up to 85% of pediatric patients who
have Crohn disease exhibit documented weight loss at the time of diagnosis, and growth failure
has been described in 15% to 40% of affected children. Although nutrient deficits are more
prevalent in patients who have extensive small bowel involvement and in those who exhibit
increased disease activity, nutritional problems are not limited to these subgroups.
In most cases, nutritional status may be improved and catch-up growth realized by enteral
alimentation (which also may serve as primary disease therapy), nutritional supplements, and
controlling symptoms, which increases appetite.
Treatment for the girl described in the vignette included the use of sulfasalazine, a known
competitive inhibitor of folate absorption. Accordingly, she is at risk of developing folic acid
deficiency. All patients receiving this drug, many of whom already may manifest reduced folate
stores, should receive dietary supplementation with folic acid to prevent a deficiency state.
Because, in part, of this property of sulfasalazine, its use in IBD largely has been supplanted by
the 5-aminosalicylate analogs, which do not interfere with folate metabolism.
Although one recent study in pediatric patients demonstrated normal folate stores at the time
of Crohn disease diagnosis, previous work in adults found both reduced folate concentrations
and impaired folate absorption. Low folate stores may result from several factors, including
reduced intake of green, leafy vegetables; increased disease activity; and drug-nutrient
interactions. The clinical consequences of this micronutrient deficit in IBD have not been fully
elucidated, but reduced folate has been associated with elevated homocysteine concentrations
in children who have both Crohn disease and ulcerative colitis, suggesting a possible link
between folate deficiency and the thrombotic complications of these disorders. Deficiencies of
vitamins B6 and B12, the latter a particular concern in patients who have extensive ileal disease,
also have been linked to elevated concentrations of homocysteine.
Iron deficiency is the most common mineral deficiency associated with Crohn disease and is
usually due to enteric blood loss. Children who have Crohn disease may exhibit reduced serum
concentrations of copper, selenium, and zinc. These mineral deficiencies appear to be related
directly to disease activity and develop as a result of both enteric loss and oxidative stress. Zinc
deficiency also has been associated with impaired metabolism of retinol-binding protein, leading
to vitamin A deficiency. However, such nutrient deficits are unlikely to develop in patients who
exhibit low disease activity indices.
Oxidative stress may lead directly to impaired vitamin A and vitamin E nutriture. In such
cases, concentrations normalize in response to control of disease activity. Vitamins A and E, as
well as vitamins D and K, are fat-soluble and require micellar solubilization for absorption.
Accordingly, nutritional adequacy may be a concern in the setting of extensive ileal disease
leading to bile salt loss and fat malabsorption.
Maintenance of skeletal health is an important objective in the care of patients who have
Crohn disease. This is especially critical in children, for whom dysregulation of calcium and
vitamin D metabolism may lead to osteopenia and inhibit skeletal growth. Problems related to

Copyright 2010 by the American Academy of Pediatrics

page 11

Dr

_F

aq

eh

2011 PREP SA on CD-ROM

calcium homeostasis have been demonstrated in those who have Crohn disease, independent
of glucocorticoid use. Studies have shown that a significant percentage of patients exhibit
increased 1,25-dihydroxyvitamin D and reduced 25-hydroxyvitamin D concentrations, probably
because of the overexpression of 1-alpha-hydroxylase in inflamed small bowel mucosa. Low
concentrations of 25-hydroxyvitamin D result in reductions in serum calcium that, in turn, lead to
increased parathormone secretion. As a consequence, 1,25-dihydroxyvitamin D concentrations
rise, stimulating bone resorption and calcium release.
Suggested reading:
Chan EP, Lichtenstein GR. Chemoprevention: risk reduction with medical therapy of inflammatory
bowel disease. Gastroenterol Clin North Am. 2006;35:675-712. DOI: 10.1016/j.gtc.2006.07.003.
Abstract available at: http://www.ncbi.nlm.nih.gov/pubmed/16952746
Heyman MB, Garnett EA, Shaikh N, et al. Folate concentrations in pediatric patients with newly
diagnosed inflammatory bowel disease. Am J Clin Nutr. 2009;89:545-550. Abstract available at:
http://www.ncbi.nlm.nih.gov/pubmed/19116333
Nakano E, Taylor CJ, Chada L, McGaw J, Powers HJ. Hyperhomocystinemia in children with
inflammatory bowel disease. J Pediatr Gastroenterol Nutr. 2003;37:586-590. Available at:
http://journals.lww.com/jpgn/Fulltext/2003/11000/Hyperhomocystinemia_in_Children_With_Inflam
matory.16.aspx

Copyright 2010 by the American Academy of Pediatrics

page 12

Dr

_F

aq

eh

2011 PREP SA on CD-ROM


Question: 216

A 15-year-old, who wishes to join the school track team, decided to test his endurance with a 3hour run. He drank water before and during the run to maintain hydration, but he fainted and was
taken to an emergency department. You advise him about fluids and nutrition and make specific
suggestions for days when he plans to run for more than 1 hour.
Of the following, the MOST appropriate advice for these days is for him to ingest

A. caffeine-containing drinks during and after long runs


B. carbohydrate drinks before, during, and after long runs
C. daily amino acid supplements
D. high-protein meals 3 to 6 hours before long runs
E. increased amounts of water after the run

Copyright 2010 by the American Academy of Pediatrics

page 13

Dr

_F

aq

eh

2011 PREP SA on CD-ROM


Critique: 216

Preferred Response: B

Optimal exercise performance requires good hydration, with attention given to fluids before,
during, and after exercise (Item C216). Fluid requirements for an athlete vary by the type of
sport; the duration of activity; the ambient temperature; and individual factors such as body
weight, genetics, and sweating rates. In addition to water, electrolyte and carbohydrate intake
becomes important when sports participation lasts longer than 1 hour. Accordingly, the boy
described in the vignette should ingest carbohydrate drinks before, during, and after long runs.
A 1% loss in body weight with exercise can result in an increased heart rate and impaired
heat transfer to the skin and environment. Loss of 1 kg of weight is equivalent to loss of 1 L of
fluid. Dehydration or loss of 2% of body weight impairs performance, cognition, short-term
memory, attention, and visual motor tracking and increases the risk for heat exhaustion and heat
stroke. Water alone is adequate for exercise that lasts 1 hour or less. Excessive water intake
puts the athlete at risk for hyponatremia, which may have been the case for the boy in the
vignette.
The recommended daily allowance for protein is 0.95 g/kg per day in 4 to 13 year olds, 0.85
g/kg per day in 14 to 18 year olds, and 0.8 g/kg per day in adults. No evidence currently
supports the need for additional dietary protein requirements in healthy adolescents who
undertake endurance or resistance exercise. Supplements of amino acids and other proteins do
not enhance performance, and long-term studies on their safety have not been conducted. A
high-carbohydrate, low-fat meal is recommended before an endurance event.
Energy drinks are marketed for their stimulant effect and may contain caffeine, vitamins, and
herbal supplements. They have the same amount of calories as standard soft drinks. Ingestion
of more than 400 mg of caffeine daily by adults has the potential to induce behavioral,
cardiovascular, bone, and reproductive effects, and ingestion of more than 100 mg by
adolescents may result in high blood pressure. Sports drinks are manufactured for those
undertaking physical activity lasting more than 1 hour at a time and contain carbohydrates and
electrolytes. These drinks reduce fatigue by maintaining hydration, electrolyte balance, and blood
glucose concentrations during endurance sports. Use should be at a rate consistent with sweat
loss. The sodium in these drinks helps with water and glucose absorption from the small
intestine and is especially important in maintaining hydration during exercise in the heat.
Suggested reading:
Jardine DS. Heat illness and heat stroke. Pediatr Rev. 2007;28:249-258. DOI:
10.1542/10.1542/pir.28-7-249. Available at:
http://pedsinreview.aappublications.org/cgi/content/full/28/7/249
Kleinman RE. Sports nutrition. In: Pediatric Nutrition Handbook. 6th ed. Elk Grove Village, Ill:
American Academy of Pediatrics; 2009:225-247
Malinauskas BM, Aeby VG, Overton RF, Carpenter-Aeby T, Barber-Heidal K. A survey of energy
drink consumption patterns among college students. Nutr J. 2007;6:35. DOI: 10.1186/1475-28916-35. Available at: http://www.nutritionj.com/content/6/1/35

Copyright 2010 by the American Academy of Pediatrics

page 14

Dr

_F

aq

eh

2011 PREP SA on CD-ROM

Rodriguez NR, DiMarco NM, Langley S; American Dietetic Association; Dietitians of Canada;
American College of Sports Medicine. Position of the American Dietetic Association, Dietitians of
Canada and the American College of Sports Medicine: nutrition and athletic performance. J Am
Diet Assoc. 2009;109:509-527. Abstract available at:
http://www.ncbi.nlm.nih.gov/pubmed/19278045
Savoca MR, Evans CD, Wilson ME, Harshfiel GA, Ludwig DA. The association of caffeinated
beverages with blood pressure in adolescents. Arch Pediatr Adolesc Med. 2004;158:473-477.
Available at: http://archpedi.ama-assn.org/cgi/content/full/158/5/473
Sawka MN, Burke LM, Eichner ER, Maughan RJ, Montain SJ, Stachenfeld NS. American College
of Sports Medicine position stand: exercise and fluid replacement. Med Sci Sports Exerc.
2007;39:377390. DOI: 10.1249/mss.0b013e31802ca597. Available at:
http://journals.lww.com/acsmmsse/Fulltext/2007/02000/Exercise_and_Fluid_Replacement.22.aspx
von Duvillard SP, Arciero PJ, Tietjen-Smith T, Alford K. Sports drinks, exercise training, and
competition. Curr Sports Med Rep. 2008;7:202-208. Abstract available at:
http://www.ncbi.nlm.nih.gov/pubmed/18607221

Copyright 2010 by the American Academy of Pediatrics

page 15

Dr

_F

aq

eh

2011 PREP SA on CD-ROM


Critique: 216

(Reprinted with permission from Kleinman RE. Sports nutrition. In: Pediatric Nutrition Handbook.
6th ed. Elk Grove Village, Ill: American Academy of Pediatrics; 2009:225-247)
Dietary recommendations before, during, and after physical exercise.

Copyright 2010 by the American Academy of Pediatrics

page 16

Dr

_F

aq

eh

2011 PREP SA on CD-ROM


Question: 232

You are evaluating a 15-year-old girl who runs track and has completed orthopedic treatment for
a stress fracture of her tibia. You note that she is shorter than other family members. She has
had a few light periods since menarche 1 year ago. Her mother states that she has always been
a picky eater, but the girl has experienced no weight loss.
Of the following, the MOST appropriate anticipatory guidance for the girl is to

A. begin combined oral contraception


B. do stretching exercises after running
C. increase her calcium intake
D. increase her overall nutritional intake
E. start daily vitamin D supplements

Copyright 2010 by the American Academy of Pediatrics

page 17

Dr

_F

aq

eh

2011 PREP SA on CD-ROM


Critique: 232

Preferred Response: D

Active individuals need proper diets to meet increased energy and fluid needs and maintain
their body weights, replenish their glycogen stores, and provide protein to build and repair
tissue. The needs depend on sex; body size; and the activity undertaken, its duration, and its
intensity. Dietary components include macronutrients (carbohydrates, protein, and fat) and
micronutrients (fluids, electrolytes, vitamins, and minerals). Most athletes increase their intake to
accommodate these increased needs. If adequate energy is ingested from a variety of foods to
maintain body weight, vitamin and mineral supplements are not needed.
Inadequate intake of energy, protein, and some micronutrients is more common among
participants in sports that have weight requirements; encourage leanness; and require clothing
that is contour-revealing, such as gymnastics, ballet, diving, cheerleading, distance running, and
weight lifting. Female athletes, who are more likely to participate in most of these activities, are at
greater risk for nutritional complications than males. Chronic energy deficits, which the girl
described in the vignette most likely is experiencing, can result in impaired growth and
development in younger athletes, loss of body weight, disruption of endocrine function, loss of
strength and endurance, increased risk for injuries (including stress fractures), and
compromised immune function. Female athletes are at risk for the female athlete triad that
includes the following three interrelated components: disordered eating, amenorrhea, and
osteoporosis. Calcium, vitamin D, and weight-bearing exercises are needed to achieve peak
bone mass in all adolescents. Calcium requirements do not change with athletic participation.
However, if intake of these micronutrients is inadequate and caloric intake is decreased, the risk
for stress fractures increases, as for this girl. Supplementation with calcium, vitamin D, or
estrogen (combined oral contraception) without increasing caloric intake does not reverse bone
changes such as osteopenia or osteoporosis. Stretching after exercise does not affect bone
health.
Suggested reading:
American Dietetic Association; Dietitians of Canada; American College of Sports Medicine,
Rodriguez NR, Di Marco NM, Langley S. American College of Sports Medicine position stand.
Nutrition and athletic performance. Med Sci Sports Exerc. 2009;41:709-731. DOI:
10.1249/MSS.0b013e31890eb86. Available at: http://journals.lww.com/acsmmsse/Fulltext/2009/03000/Nutrition_and_Athletic_Performance.27.aspx
Kleinman RE. Sports nutrition. In: Pediatric Nutrition Handbook. 6th ed. Elk Grove Village, Ill:
American Academy of Pediatrics; 2009:225-247
Lerand SJ Williams JF. The female athlete triad. Pediatr Rev. 2006;27:e12-e13. DOI:
10.1542/10.1542/pir.27-1-e12. Available at:
http://pedsinreview.aappublications.org/cgi/content/full/27/1/e12
Petrie HJ, Stover EA, Horswill CA. Nutritional concerns for the child and adolescent competitor.
Nutrition. 2004;20:620-631. DOI: 10.1016/j.nut.2004.04.002. Abstract available at:

Copyright 2010 by the American Academy of Pediatrics

page 18

Dr

_F

aq

eh

2011 PREP SA on CD-ROM

http://www.ncbi.nlm.nih.gov/pubmed/15212744
United States Department of Agriculture. Nutrition and Your Health: Dietary Guidelines for
Americans 2005. Part D: Science Base. Available at:
http://www.health.gov/dietaryguidelines/dga2005/report/HTML/D1_Adequacy.htm

Copyright 2010 by the American Academy of Pediatrics

page 19

Dr

_F

aq

eh

2010 PREP SA on CD-ROM


Question: 1

You are counseling the mother of a 3-month-old breastfed infant whose family has been urging
her to introduce cereals to her babys diet. She asks your advice.
Of the following, the MOST likely outcome of introducing solid foods at this age is to

A. accelerate the development of oral-motor skills


B. help the infant sleep through the night
C. increase the risk of food allergies
D. increase the risk of gastroesophageal reflux
E. increase the risk of gastrointestinal infections

Copyright 2010 by the American Academy of Pediatrics

page 1

Dr

_F

aq

eh

2010 PREP SA on CD-ROM


Critique: 1

Preferred Response: E

The most likely consequence of early (before 6 months of age) feeding of complementary
foods such as cereals to breastfed infants is an increased likelihood of gastrointestinal infection.
The direct relationship between early complementary feedings and the incidence of diarrheal
illness is based on several case-control studies. In one investigation from Belarus, a large group
of infants who were exclusively breastfed for more than 6 months was compared with a group
receiving a mixed diet of human milk plus solids, with solids introduced between 3 and 6 months
of age. Exclusively breastfed infants had a significantly reduced risk of one or more
gastrointestinal illnesses. Furthermore, other observations suggest that this effect may be
enhanced with greater duration and exclusivity of breastfeeding. However, prior studies have
failed to show any clear risk reduction in the prevalence of upper and lower respiratory tract
illnesses, asthma, and otitis media among exclusively breastfed infants compared with infants
who received a mixed diet of human milk and solids.
No available evidence supports the hypothesis that the introduction of solid foods either
accelerates the development of oral-motor skills or helps infants to sleep through the night.
Data concerning the effect of early introduction of solids on the development of allergies are
conflicting. The Belarus study found no reduction in risk for atopic eczema in exclusively
breastfed infants; a Finnish investigation showed a reduced eczema risk at 1 year but not at 5
years of age in a similar group. Although the Finnish study demonstrated a small reduction in any
atopic condition for exclusively breastfed infants, the results were not statistically significant.
Evidence also failed to demonstrate that early solid food introduction was associated with an
increased incidence of positive skin prick tests.
Results of obesity studies also are inconclusive. In exclusively breastfed infants, solid food
introduction prior to 6 months of age generally is associated with reduced human milk intake
without accelerated weight gain. However, formula-fed infants may be encouraged to consume
the same amount of formula, even after complementary feedings are introduced. This practice
may lead to increased calorie consumption and excessive weight gain.
Gastroesophageal reflux (GER) is the result of transient relaxations of the lower esophageal
sphincter. Studies using intraesophageal pH probe monitoring data have shown that the reflux
index (RI) (percent time that esophageal pH is less than 4) is significantly greater in infants (RI
mean upper limit of normal: ~12) than in older individuals (mean: ~6). The addition of solids to the
diet does not influence the time to resolution of clinical GER during infancy, although the
frequency and severity of symptomatic reflux episodes may be reduced, at least in part, by
thickening feedings or increasing solid consumption in appropriately aged infants.
The appropriate timing for introducing solid foods to the infant diet depends on development
of both neuromuscular function and gastrointestinal maturation. The American Academy of
Pediatrics supports exclusive breastfeeding for the first 6 postnatal months. However, from a
developmental perspective, term infants often are capable of accepting solids (complementary
foods) between 4 and 6 months of age. Maturational readiness to tolerate complementary
feedings is indicated by loss of the extrusion reflex (usually by 4 months) and by the ability to
swallow non-liquid foods. The most obvious risk posed by solid food consumption prior to
reaching these developmental milestones is that failure to achieve oropharyngeal coordination
for consuming solids may lead to aspiration.

Copyright 2010 by the American Academy of Pediatrics

page 2

Dr

_F

aq

eh

2010 PREP SA on CD-ROM

References:
Braganza SF, Adam HF. In brief: gastroesophageal reflux. Pediatr Rev. 2005;26:304-305.
Available at: http://pedsinreview.aappublications.org/cgi/content/full/26/8/304
Fein SB, Labiner-Wolfe J, Scanlon KS, Grummer-Strawn LM. Selected complementary feeding
practices and their association with maternal education. Pediatrics. 2008;122:S91-S97.
Available at:
http://pediatrics.aappublications.org/cgi/reprint/122/Supplement_2/S91?maxtoshow=&HITS=10&h
its=10&RESULTFORMAT=&fulltext=Selected+complementary+feeding+practices+and+their+asso
ciation+with+maternal+education&searchid=1&FIRSTINDEX=0&sortspec=relevance&resourcetyp
e=HWCIT
Kleinman RE. Complementary feeding. In: Pediatric Nutrition Handbook. 5th ed. Elk Grove
Village, Ill: American Academy of Pediatrics; 2004:103-115
Kramer MS, Chalmers B, Hodnett ED, et al. Promotion of Breastfeeding Intervention Trial
(PROBIT): a randomized trial in the Republic of Belarus. JAMA. 2001;285:413-420. Available at:
http://jama.ama-assn.org/cgi/content/full/285/4/413

Copyright 2010 by the American Academy of Pediatrics

page 3

Dr

_F

aq

eh

2010 PREP SA on CD-ROM


Question: 5

During the health supervision visit for a 14-year-old boy, you note that his body mass index
(BMI) is at the 95th percentile. At last years health supervision visit, his BMI was at the 85th
percentile.
Of the following, the complication for which this boy is at the HIGHEST risk is

A. atrial fibrillation
B. hypothyroidism
C. narcolepsy
D. social isolation
E. steatorrhea

Copyright 2010 by the American Academy of Pediatrics

page 4

Dr

_F

aq

eh

2010 PREP SA on CD-ROM


Critique: 5

Preferred Response: D

Childhood obesity is one of the most common chronic conditions of childhood. It is believed
to be strongly associated with a number of morbidities, including type 2 diabetes mellitus,
hypertension, the metabolic syndrome, and psychosocial conditions. For individual children, the
immediate psychosocial effects of social isolation, discrimination, and peer problems may
accompany childhood obesity. Obese adolescents have been reported to have lower selfesteem as well as increased rates of sadness, social isolation, loneliness, and nervousness.
Several studies have demonstrated the relationship between obesity, as measured by body
mass index (BMI) and the psychosocial development of children.
Adolescent obesity is a strong predictor of adult obesity, and adult obesity has been
associated with depression, especially in women. Studies also have suggested an association
between depression in adolescence and higher BMI in adulthood. Whether depression leads to
obesity or obesity causes depression is unclear. In contrast, there is a strong association
between lower self-esteem and higher BMI across the elementary school years. In many
children, the presence of an increased BMI and obesity precedes low self-esteem, suggesting a
causal relationship. Accordingly, prevention and management strategies for children who are
overweight and obese should be undertaken early to minimize the impact on self-esteem and the
other important psychosocial aspects of healthy development.
The boy described in the vignette, who meets the diagnostic criteria for obesity (BMI >95th
percentile), may be at increased risk for obstructive apnea and resulting effects on the right
ventricle, but he is not necessarily at a greater risk for atrial arrhythmias such as atrial fibrillation.
Similarly, he may have poor sleeping habits, but obesity does not lead to narcolepsy. Although
hypothyroidism can be associated with weight gain, no evidence suggests that obesity leads to
hypothyroidism. Finally, obesity is not known to be a cause of steatorrhea.
References:
Goodman E, Whitaker RC. A prospective study of the role of depression in the development and
persistence of adolescent obesity. Pediatrics. 2002;110:497-504. Available at:
http://pediatrics.aappublications.org/cgi/content/full/110/3/497
Hesketh K, Wake M, Waters E. Body mass index and parent-reported self-esteem in elementary
school children: evidence for a causal relationship. Int J Obes Relat Metab Disord.
2004;28:1233-1237. Available at: http://www.nature.com/ijo/journal/v28/n10/full/0802624a.html
Ludwig DS. Childhood obesitythe shape of things to come. N Engl J Med. 2007;357:2325-2327.
Available at: http://content.nejm.org/cgi/content/full/357/23/2325
Stunkard AJ, Wadden TA. Psychological aspects of severe obesity. Am J Clin Nutr. 1992;55(2
suppl):524S-532S. Available at: http://www.ajcn.org/cgi/reprint/55/2/524S

Copyright 2010 by the American Academy of Pediatrics

page 5

Dr

_F

aq

eh

2010 PREP SA on CD-ROM


Question: 17

During a routine health supervision visit, the mother of a 2 month-old male infant tells you that
the baby has been experiencing bloating and flatulence. His diet consists of 5 to 6 oz of a cow
milk-based formula given five times per 24 hours. Because of frequent spitting-up, his mother
recently added rice cereal to each bottle. He has two to three seedy stools per day. On physical
examination, the baby is alert and vigorous. His length and weight are tracking between the 50th
and 75th percentiles. The infants mother asks you whether switching to a soy protein-based
formula will help her babys "gassiness."
Of the following, the MOST likely the cause of this infants symptoms is

A. cow milk protein allergy


B. excessive energy intake
C. incomplete starch digestion
D. lactose malabsorption
E. sucrase-isomaltase deficiency

Copyright 2010 by the American Academy of Pediatrics

page 6

Dr

_F

aq

eh

2010 PREP SA on CD-ROM


Critique: 17

Preferred Response: C

The infant described in the vignette has been given formula thickened with rice cereal to
ameliorate spitting-up. Following the introduction of cereal, his mother has noted increased
"gassiness." The most likely cause of this symptom is incomplete starch digestion.
Development of the digestive-absorptive function of the gastrointestinal tract is not complete
at birth. The newborn can assimilate considerable amounts of complex carbohydrates through
hydrolysis by salivary gland amylase until pancreatic function and small intestinal intraluminal
pancreatic amylase activity mature. Nevertheless, until pancreatic maturity is achieved, and
certainly in infants younger than 4 months of age, dietary starches may be hydrolyzed
incompletely. As a result, increased amounts of undigested carbohydrate pass into the colon,
where bacterial fermentation results in gas production that may cause the symptoms described
for the infant in the vignette.
A diagnosis of cow milk protein allergy frequently is considered in the differential diagnosis
of a variety of diverse gastrointestinal complaints. Symptoms that may be associated with cow
milk protein intolerance include diarrhea, failure to thrive, hypoproteinemia, hematochezia,
anemia, and vomiting as well as other cutaneous and systemic manifestations of atopy. The
relationship between infantile colic and cow milk protein allergy remains highly controversial,
particularly when fussiness or irritability is the sole complaint. For a thriving infant who develops
vague gastrointestinal symptoms after the type of dietary changes described in the vignette,
cow milk protein allergy should be considered only after ruling out other, more likely causes,
such as incomplete digestion of complex carbohydrates.
It is unlikely that the infant described in the vignette has excessive energy intake because
his weight gain is not excessive, and thickening of the formula does not appreciably add to
energy intake in an infant who is consuming 25 to 30 oz of formula per day.
Lactase concentrations reach mature values in the small intestine by the 36th week of
gestation in all healthy infants. Congenital or early-onset primary lactose intolerance is an
extremely rare condition that is associated with severe diarrhea and inanition. It typically
presents with voluminous diarrhea soon after the first feedings of human milk or cow milk-based
formula. During infancy and childhood, secondary lactase deficiency may occur as a
consequence of intestinal mucosal damage following a prolonged diarrheal illness, as a result of
other intestinal disorders (eg, celiac disease), or in association with malnutrition.
Sucrase-isomaltase (SI) deficiency is the most common congenital disaccharidase
deficiency. Diarrhea is a virtually universal symptom of SI deficiency and may be associated
with poor weight gain. Symptoms usually appear in older infants following the introduction of
sucrose-containing foods, particularly fruits and juices. Infants who have SI deficiency also do
not tolerate soy or protein hydrolysate formulas because both sucrose and glucose polymers
are maldigested and malabsorbed.
References:
Craig WR, Hanlon-Dearman A, Sinclair C, Taback S, Moffatt M. Metoclopramide, thickened
feedings, and positioning for gastro-oesophageal reflux in children under two years. Cochrane
Database Syst Rev. 2004;3:CD003502. Available at:

Copyright 2010 by the American Academy of Pediatrics

page 7

Dr

_F

aq

eh

2010 PREP SA on CD-ROM

http://www.mrw.interscience.wiley.com/cochrane/clsysrev/articles/CD003502/frame.html
Hall RT, Carroll RE. Infant feeding. Pediatr Rev. 2000; 21:191-200. Available at:
http://pedsinreview.aappublications.org/cgi/content/full/21/6/191
Montes RG. Carbohydrate malabsorption. In: Rudolph CD, Rudolph AM, Hostetter MK, Lister G,
Siegel NJ, eds. Rudolphs Pediatrics. 21st ed. New York, NY: McGraw-Hill; 2003:1423-1427
Thomas DW, McGilligan K, Eisenberg LD, Lieberman HM, Rissman EM. Infantile colic and type of
milk feeding. Am J Dis Child. 1987;141:451-453. Abstract available at:
http://www.ncbi.nlm.nih.gov/pubmed/3494394

Copyright 2010 by the American Academy of Pediatrics

page 8

Dr

_F

aq

eh

2010 PREP SA on CD-ROM


Question: 20

A nurse practitioner in your clinic has asked you to review the chart of an 11-year-old boy
referred from school for evaluation of acanthosis nigricans.
Of the following, the physical examination parameter that is MOST likely to be useful in predicting
comorbidities in this patient is

A. blood pressure at the 75th percentile for height


B. body mass index at the 95th percentile for age
C. sum of triceps and subscapular skinfold thickness greater than 90% for age
D. weight at the 50th percentile and height at the 25th percentile for age
E. weight at the 95th percentile and height at the 75th percentile for age

Copyright 2010 by the American Academy of Pediatrics

page 9

Dr

_F

aq

eh

2010 PREP SA on CD-ROM


Critique: 20

Preferred Response: B

Because acanthosis nigricans is associated with obesity, the young man in the vignette is
likely to be overweight and at risk for numerous comorbidities, including hypertension,
hyperlipidemia, and the metabolic syndrome.
Although several methods have been used to assess obesity, body mass index (BMI),
defined as weight in kilograms divided by height in meters squared (kg/m2), has become the
standard measure for children, adolescents, and adults. The correlation of BMI with measures of
adiposity is excellent in adults, but slightly less reliable for children, whose BMIs change with
age during childhood and adolescence. Of note, boys have less body fat than do girls at the
same BMI. Total body fat correlates with sexual maturational level more than age because as
sexual maturity progresses, body fat increases. Those who have a higher waist-to-hip ratio at
the same BMI have more body fat.
In 2000, the Centers for Disease Control and Prevention (CDC) released new growth
curves, including BMI curves for boys and girls from ages 2 to 20 years
(www.cdc.gov/growthcharts). Obesity has been defined as a BMI greater than the 95th
percentile for age on the 2000 CDC growth curves. Children whose BMIs fall between the 85th
and 95th percentiles for age are considered overweight. Children whose BMIs are greater than
the 99th percentile are defined as severely obese.
Weight-for-height is a less accurate measure and is subject to misinterpretation of
overweight versus obesity. In the estimation of risk for comorbid conditions, including diabetes
or insulin insensitivity, hypertension, hyperlipidemia, or metabolic syndrome, the BMI now
represents a standardized measurement. However, high weight-for-height must be used to
assess obesity in children younger than 2 years of age for whom BMI norms are not available. It
is unclear whether childhood obesity is an independent risk factor for hyperlipidemia and
atherosclerotic heart disease. It is known that the higher the BMI, the greater the risk of left
ventricular wall thickness in children who have hypertension and that BMI correlates with
arterial wall thickness. Skinfold thickness measurements are accurate in estimating total body fat
but are not currently the standard for determining risk for comorbidities of obesity. In addition,
they may be difficult to obtain in the primary care office setting due to issues with interobserver
reliability, training of staff in the technique, and the expense of the calipers ($200). However,
skinfold thickness measurements may be useful in very athletic, muscular adolescent males
whose BMIs are artificially elevated by high muscle mass.
All children older than 3 years of age should have blood pressures measured at each health
supervision visit; children who are overweight should have blood pressures monitored more
frequently. A blood pressure that is at the 75th percentile is within the normal range and less
likely to be associated with comorbidities than BMI at the 95th percentile.
References:
Barlow SE and the Expert Committee. Expert Committee recommendations regarding the
prevention, assessment, and treatment of child and adolescent overweight and obesity:
summary report. Pediatrics. 2007;120 (suppl):S164-S192. Available at:
http://pediatrics.aappublications.org/cgi/content/full/120/Supplement_4/S164

Copyright 2010 by the American Academy of Pediatrics

page 10

Dr

_F

aq

eh

2010 PREP SA on CD-ROM

Feld LG, Corey H. Hypertension in childhood. Pediatr Rev. 2007;28:283-298. Available at:
http://pedsinreview.aappublications.org/cgi/content/full/28/8/283
Freedman DS, Kettel Khan L, Mei A, Dietz WH, Srinivasan SR, Berenson GS. Relation of
childhood height to obesity among adults: the Bogalusa Heart Study. Pediatrics. 2002;109:e23.
Available at: http://pediatrics.aappublications.org/cgi/content/full/109/2/e23
Katzmarzyk PT, Srinivasan SR, Chen W, Malina RM, Bouchard C, Berenson GS. Body mass
index, waist circumference, and clustering of cardiovascular disease risk factors in a biracial
sample of children and adolescents. Pediatrics. 2004;114:e198-e205. Available at:
http://pediatrics.aappublications.org/cgi/content/full/114/2/e198
Schneider MB, Brill SR. Obesity in children and adolescents. Pediatr Rev. 2005;26:155-162.
Available at: http://pedsinreview.aappublications.org/cgi/content/full/26/5/155
Sorof J, Daniels S. Obesity hypertension in children: a problem of epidemic proportion.
Hypertension. 2002;40:441-447. Available at:
http://hyper.ahajournals.org/cgi/content/full/40/4/441
Summerbell CD, Waters E, Edmunds LD, Kelly S, Brown T, Campbell KJ. Interventions for
preventing obesity in children. Cochrane Database Syst Rev. 2005;3:CD001871
Whitlock EP, OConnor EA, Williams SB, Beil TL, Lutz KKW. Effectiveness of Weight Management
Programs in Children and Adolescents. Evidence Report/Technology Assessment No. 170.
AHRQ Publication No. 08-E014. Rockville, Md: Agency for Healthcare Research and Quality, U.S.
Department of Health and Human Services; 2008. Available at:
http://www.ncbi.nlm.nih.gov/books/bv.fcgi?rid=hstat1b.chapter.139937

Copyright 2010 by the American Academy of Pediatrics

page 11

Dr

_F

aq

eh

2010 PREP SA on CD-ROM


Question: 33

You are evaluating a 2-year-old daughter of strict vegan parents. Her birthweight at term was
3.5 kg. Since weaning at 12 months of age, the childs diet has included a homemade,
macrobiotic-based formula. In your office today, the girls weight is 11.2 kg.
Of the following, the childs diet MOST likely is deficient in

A. essential amino acids


B. linoleic acid
C. vitamin A
D. vitamin B12
E. vitamin C

Copyright 2010 by the American Academy of Pediatrics

page 12

Dr

_F

aq

eh

2010 PREP SA on CD-ROM


Critique: 33

Preferred Response: D

Strict vegan diets include foods that come solely from plant sources. Such diets generally
contain adequate amounts of vitamins A and C as well as essential fatty acids (including linoleic
acid). However, a strict vegan diet instituted after weaning contains very little vitamin B12, a
nutrient primarily found in meats, eggs, and dairy products, unless supplements are provided.
Breastfed infants of vegan mothers may develop vitamin B12 deficiency, but only if maternal
stores are low. Although the vitamin composition of human milk is directly related to dietary intake
of vitamins A, C, D, and the B group, studies comparing the vitamin content of human milk from
vegan compared with nonvegan mothers have not demonstrated any significant micronutrient
differences. Commercial soy-based formulas are alternatives for vegan mothers who do not
breastfeed. In most cases, therefore, the greatest potential nutritional risks for both breastfed
and soy formula-fed infants of vegan parents occur after weaning. This is particularly the case
when a homemade weaning formula is given. Conversely, commercially available soy milks are
supplemented with vitamins.
Studies in both the United States and the United Kingdom have shown that vegan children
exhibit small but significant differences in growth variables (height and weight percentiles)
compared with children eating mixed diets. This observation most likely is the consequence of
group differences in total energy consumption, although other studies have demonstrated that
the calcium and zinc content of the vegan diet also may be low, indicating the requirement for
supplementation. Conversely, the essential amino acid and total protein intake of vegan children
has been shown to be adequate to support normal growth. Despite the lower mean height and
weight of vegan children compared with children eating mixed diets, a large British study found
no evidence of growth failure (weight or height less than the 5th percentile) in vegan children,
and no between-group differences were noted in terms of muscle strength and overall health.
A routine health assessment of any child should include a careful dietary history. Information
about specific cultural or family customs permits identification of patients at nutritional risk and
aids the clinician in determining whether caregivers require education regarding appropriate
nutrition for growing children. For example, in industrialized countries, vitamin D deficiency
rickets is an emerging nutritional problem. Such deficiency is particularly prevalent for darkskinned infants living in temperate or northern climates, those whose cultural/religious customs
may include extensive covering of body surfaces, and in infants and children who receive little
direct sunlight exposure (ie, less than 30 minutes to the face and hands three times per week).
References:
Graham EA. Economic, racial and cultural influences on the growth and maturation of children.
Pediatr Rev. 2005;26:290-294. Available at:
http://pedsinreview.aappublications.org/cgi/content/full/26/8/290
Hebbelinck M, Clarys P, De Malsche A. Growth, development, and physical fitness of Flemish
vegetarian children, adolescents, and young adults. Am J Clin Nutr. 1999;70(3 suppl):579S585S. Available at: http://www.ajcn.org/cgi/content/full/70/3/579S

Copyright 2010 by the American Academy of Pediatrics

page 13

Dr

_F

aq

eh

2010 PREP SA on CD-ROM

Joiner TA, Foster C, Shope T. The many faces of vitamin D deficiency rickets. Pediatr Rev.
2000;21:296-302. Available at: http://pedsinreview.aappublications.org/cgi/content/full/21/9/296
Kramer MS, Guo T, Platt RW, et al. Breastfeeding and infant growth: biology or bias? Pediatrics.
2002;110:343-347. Available at: http://pediatrics.aappublications.org/cgi/content/full/110/2/343
Moilanen BC. In brief: vegan diets in infants, children, and adolescents. Pediatr Rev. 2004;25:174176. Available at: http://pedsinreview.aappublications.org/cgi/content/full/25/5/174
Sanders TA. Vegetarian diets and children. Pediatr Clin North Am. 1995;42:955-965. Abstract
available at: http://www.ncbi.nlm.nih.gov/pubmed/7610022

Copyright 2010 by the American Academy of Pediatrics

page 14

Dr

_F

aq

eh

2010 PREP SA on CD-ROM


Question: 49

The mother of a 5-month-old boy has come to your office seeking nutritional advice. She
exclusively breastfed the infant for the first 4 months, then weaned the baby to a standard, cow
milk protein-based infant formula. One week after weaning, she noted that the baby "strained
with stool." Because of her concerns regarding the development of constipation, the mother
switched him to a formula containing 2 mg/L iron.
Of the following, the MOST important dietary recommendation for this infant is to

A. add pureed vegetables to the diet


B. change to a cow milk protein-based formula containing 12 mg/L iron
C. change to a soy protein-based formula
D. continue the present regimen and supplement with 4 oz/day diluted apple juice
E. substitute oatmeal for rice cereal in the diet

Copyright 2010 by the American Academy of Pediatrics

page 15

Dr

_F

aq

eh

2010 PREP SA on CD-ROM


Critique: 49

Preferred Response: B

For the infant described in the vignette, the most appropriate dietary recommendation is to
return the infant to a formula fortified with 12 mg/L iron. Parents frequently use low-iron
formulas in the false belief that iron-supplemented formulas induce constipation. However, the
change to a low iron-containing formula not only fails to alter the stooling pattern but can place
the infant at risk for iron deficiency.
Although the optimal iron content of infant formula has been debated, all available data
indicate that iron supplementation is essential for formula-fed infants to prevent a deficiency
state. For nursing infants, the lactoferrin content of human milk greatly enhances iron
bioavailability and absorption, despite the milks low iron content. Therefore, during the first few
months, no added iron is required. After 6 months, however, breastfed infants are at risk for
iron deficiency, unless dietary supplements are provided. For infants weaned to the bottle, this
may be accomplished by using a standard iron-fortified formula. For infants who continue to
nurse after 6 months, iron-fortified stage 1 baby foods and cereals meet the requirement.
Supplementing human milk with formula or changing formulas are common responses to a
wide range of nonspecific complaints. In otherwise healthy and thriving infants, available
evidence neither supports this practice nor recommends the consumption of solids for other
than nutritionally indicated purposes.
In situations such as that described in the vignette, caregivers often try a soy formula,
usually on the advice of friends or family. In fact, if constipation is the perceived issue, soy
formulas actually may exacerbate the problem. Proprietary soy preparations contain either
sucrose or glucose polymers as the constituent carbohydrate. Such di- or polysaccharides may
be absorbed more efficiently compared with lactose, particularly in young infants, resulting in a
firmer stool. From a nutritional perspective, all commercial soy formulas are iron-fortified. Such
fortification is required for soy-based formulas because soy protein products contain phytates,
which bind iron and other minerals intraluminally.
The consumption of poorly absorbed carbohydrates (eg, sorbitol in fruit juice) or fiber (fruits,
vegetables, cereals) may aid in alleviating true constipation, but the infant described in the
vignette is not suffering from constipation, and little direct evidence supports such dietary
interventions. In addition, supplementing the infants diet with fruit juice may lead to excessive
weight gain due to increased intake of sugars.
Parents should be reassured that straining with the passage of a soft stool is common
during the first few postnatal months. Discomfort prior to and during defecation results from the
infants inability to coordinate pelvic floor relaxation with the Valsalva maneuver and to
straighten the anal canal when lying down. Unless straining is accompanied by passage of hard
stools, intervention is not indicated and does not ameliorate the problem.
References:
Abi-Hanna A, Lake AM. Constipation and encopresis in childhood. Pediatr Rev. 1998;19:23-31.
Available at: http://pedsinreview.aappublications.org/cgi/content/full/19/1/23
American Academy of Pediatrics, Committee on Nutrition. Iron fortification of infant formulas.

Copyright 2010 by the American Academy of Pediatrics

page 16

Dr

_F

aq

eh

2010 PREP SA on CD-ROM

Pediatrics. 1999;104:119-123. Available at:


http://pediatrics.aappublications.org/cgi/content/full/104/1/119
Dee DL, Sharma AJ, Cogswell ME, Grummer-Strawn LM, Fein SB, Scanlon KS. Sources of
supplemental iron among breastfed infants during the first year of life. Pediatrics. 2008;122:S98S104. Available at: http://pediatrics.aappublications.org/cgi/content/full/122/Supplement_2/S98
Hall RT, Carroll RE. Infant feeding. Pediatr Rev. 2000; 21:191-200. Available at:
http://pedsinreview.aappublications.org/cgi/content/full/21/6/191
McQueen DA. In brief: the nutritional adequacy of mineral content of formulas. Pediatr Rev.
1997;18:67-69. Available at: http://pedsinreview.aappublications.org/cgi/content/full/18/2/67
Pizarro F, Yip R, Dallman PR, Olivares M, Hertrampf E, Walter T. Iron status with different infant
feeding regimens: relevance to screening and prevention of iron deficiency. J Pediatr.
1991;118:687-692. Abstract available at: http://www.ncbi.nlm.nih.gov/pubmed/2019922
Scariati PD, Grummer-Strawn LM, Fein SB, Yip R. Risk of diarrhea related to iron content of
infant formula: lack of evidence to support the use of low-iron formula as a supplement for
breastfed infants. Pediatrics. 1997;99:e2. Available at:
http://pediatrics.aappublications.org/cgi/content/full/99/3/e2

Copyright 2010 by the American Academy of Pediatrics

page 17

Dr

_F

aq

eh

2010 PREP SA on CD-ROM


Question: 65

A 3-year-old boy who has Noonan syndrome presents with diarrhea and poor weight gain of
several months duration. One of your cardiology colleagues is treating the boy for pulmonary
valvular stenosis. On physical examination, you note abdominal distention and pitting edema of
both lower extremities (Item Q65) and the presacral area.
Of the following, the test MOST likely to identify the cause of this boys problem is

A. abdominal ultrasonography
B. echocardiography
C. fecal alpha-1-antitrypsin measurement
D. percutaneous liver biopsy
E. serum albumin measurement

Copyright 2010 by the American Academy of Pediatrics

page 18

Dr

_F

aq

eh

2010 PREP SA on CD-ROM


Question: 65

Pitting edema (arrow), as described for the boy in the vignette. (Reprinted with permission from
Cavanaugh RM Jr. Orthostatic edema in adolescents: more than just walking on water. Pediatr
Rev. 2005;26:115-124.)

Copyright 2010 by the American Academy of Pediatrics

page 19

Dr

_F

aq

eh

2010 PREP SA on CD-ROM


Critique: 65

Preferred Response: C

Congenital cardiac disease, including atrioventricular canal defects and right-sided heart
lesions, frequently is associated with Noonan syndrome. The valvular pulmonic stenosis
described for the boy in the vignette can lead to increased right heart pressures and may result
in heart failure. Subsequent elevations in lymphatic pressure cause both hepatic congestion and
intestinal lymphangiectasis, leading to "weeping" of protein into the intestinal lumen. Excessive
intestinal protein loss results in hypoproteinemia and edema. In addition to such secondary
lymphangiectasia, other lymphatic abnormalities, including primary intestinal lymphangiectasia (ie,
without associated heart disease), have been identified in up to 20% of children affected by
Noonan syndrome.
The diagnosis of such primary and secondary protein-losing enteropathies may be
established readily by measuring fecal concentrations of alpha-1-antitrypsin, a stable and
nondietary protein that is a sensitive and specific marker of intestinal protein loss.
Typically, clinical signs of hypoproteinemia may appear when the serum albumin measures
less than 2.5 g/dL (25 g/L). Although this measurement can identify hypoproteinemia, it cannot
differentiate among the varied causes of protein loss, which may involve disorders associated
with increased urinary losses, hepatic synthesis, systemic catabolism with increased albumin
degradation, gastrointestinal secretion, inflammation, and lymphatic obstruction. In the absence
of signs of liver failure or evidence of proteinuria, gastrointestinal protein loss should be
suspected. A protein-losing enteropathy not only may result from cardiovascular dysfunction
with secondary intestinal lymphangiectasia, but its source may be a gastrointestinal
malabsorptive or inflammatory state. Abdominal ultrasonography, echocardiography,
percutaneous liver biopsy, and serum albumin measurement may be useful in defining specific
conditions associated with hypoproteinemia but are not useful in establishing its cause.
As stated previously, protein loss across the intestinal mucosa can result from a wide
variety of both gastrointestinal and nongastrointestinal illnesses. Hypoalbuminemia and edema
develop when enteric losses exceed hepatic albumin synthetic capability. The list of disorders
that may be associated with a protein-losing enteropathy is large (Item C65). Secondary
lymphangiectasia is a well-described complication of both right- and left-sided heart failure and
can be manifested both by intestinal and pulmonary protein loss. The latter may lead to pleural
effusion and chylothorax and can occur with or without an associated protein-losing
enteropathy. The most common clinical scenario leading to this problem is following the Fontan
procedure for hypoplastic left heart syndrome. However, any cardiac lesion or cardiac surgical
procedure that results in elevated right-sided pressures, including pulmonary valvular stenosis
(a common cardiac lesion in Noonan syndrome), may be the precipitating factor.
For most of the disorders associated with protein-losing enteropathy, gastrointestinal loss is
the sole contributor to the hypoproteinemic state, although some are characterized by lymphatic
obstruction and protein losses at other sites. Thus, primary intestinal lymphangiectasia often is
associated with thoracic duct obstruction leading to chylothorax as well as with peripheral
lymphedema. Intestinal protein excretion, in the absence of lymphatic losses elsewhere, can
result from mechanical obstruction of lymphatic channels (eg, malrotation, malignancy) and
diverse gastrointestinal and vascular inflammatory conditions. One interesting yet uncommon
disorder in childhood is hypertrophic rugal gastropathy or Mntrier disease. This generally

Copyright 2010 by the American Academy of Pediatrics

page 20

Dr

_F

aq

eh

2010 PREP SA on CD-ROM

benign, self-limited illness requires endoscopy and gastric biopsy for diagnosis, although
findings of giant rugal folds on upper gastrointestinal radiographic series may be suggestive.
The disorder has been linked to infection with cytomegalovirus, but other autoimmune and toxic
insults have been proposed.

Copyright 2010 by the American Academy of Pediatrics

page 21

Dr

_F

aq

eh

2010 PREP SA on CD-ROM


Question: 81

You are asked to write intravenous nutrition orders for a 5-year-old, 20-kg boy, who was
admitted to the hospital for treatment of presumed idiopathic pancreatitis. Based on the childs
clinical status, you expect him to take nothing by mouth for 5 to 7 days.
Of the following intravenous regimens, the one that is MOST appropriate is

A. central venous alimentation with 20% dextrose and 1.5% amino acids
B. central venous alimentation with 20% dextrose, 1.5% amino acids, and 20 g/day lipid
C. peripheral venous infusion with 10% dextrose and 3.0% amino acids
D. peripheral venous infusion with 10% dextrose, 1.5% amino acids, and 10 g/day lipid
E. peripheral venous infusion with 15% dextrose and 2.5% amino acids

Copyright 2010 by the American Academy of Pediatrics

page 22

Dr

_F

aq

eh

2010 PREP SA on CD-ROM


Critique: 81

Preferred Response: D

Decisions regarding nutrition support of the hospitalized patient must be based on the
answers to three critical questions:
1.Can the gastrointestinal (GI) tract be used, either partially or totally?
2.If parenteral nutrition is required, for how long will it be used?
3.Based upon the answers to questions 1 and 2, how may the patients nutrient and fluid
requirements best be met?
The child described in the vignette, who has acute pancreatitis, is expected to require
parenteral nutrition for 5 to 7 days. The parenteral route of administration is chosen both to
replace fluids lost due to pancreatic inflammation and to promote "pancreatic rest" by decreasing
the cephalic, gastric, and intestinal phases of pancreatic secretion. Because the length of
parenteral infusion is estimated to be for 7 or fewer days, a peripheral vein is the indicated route
of administration. Peripheral intravenous nutrition comprising 10% dextrose, 1.5% amino acids,
and 10 g/day lipid is best for supporting this childs basal energy and protein needs and
ameliorating the potential catabolic state during recovery.
Parenteral nutrition can be a lifesaving intervention, providing essential fluids and nutrients
for the patient in whom the GI tract cannot be used fully to meet nutritional needs for extended
periods. Parenteral nutrition administered via central venous catheter generally should be
reserved for two situations. The first is when no oral/enteral nutrition may be given for more
than 7 days because of either medical or postsurgical conditions. The second is when partial
oral/enteral feedings, in conjunction with peripheral intravenous nutrition, either cannot meet
nutrient needs or will be required for extended periods, thus presenting problems of prolonged
peripheral venous access. Central venous support also may be required in some children when
venous access problems make peripheral infusions difficult to maintain, either because of patient
age or clinical condition. Whenever possible, the GI tract should be used to deliver nutrients,
even if only a portion of total daily requirements for energy and protein are met. Infants deprived
of oral feeding stimulation for long periods may lose the desire or the ability to feed. Intraluminal
nutrients are essential for maintaining intestinal function. Prolonged absence of oral/enteral
feedings also may compromise the mucosal barrier and increase the risk for bacteria and
bacterial products in the intestine entering the blood.
Although a comprehensive review of fluid and macro- and micronutrient needs for infants
and children is beyond the scope of this critique, daily requirements for a child weighing 20 kg or
more who is receiving parenteral nutrition in the hospital include:
1.Fluid = 1,500 mL + 20 mL/kg for more than 20 kg
2.Energy = 30 to 75 kcal/kg as dextrose (3.4 kcal/g) (~10 to 20 g/kg)
3.Lipids = 1 to 3 g/kg
4.Amino Acids = 1 to 2 g/kg
For the patient receiving total intravenous nutrition via central venous catheter, dextrose
concentrations generally are in the range of 20%. However, solutions containing more than
12.5% dextrose are highly sclerosing to small veins and should be avoided in all peripheral
infusions. In most cases, a 10% dextrose solution is employed for peripheral vein infusions.
For the 20-kg boy described in the vignette, total fluid volume initially should be in the range
Copyright 2010 by the American Academy of Pediatrics

page 23

Dr

_F

aq

eh

2010 PREP SA on CD-ROM

of 1,500 mL per 24 hours. An infusate containing 10% dextrose provides 150 g of dextrose to
this child, which is equal to 510 kcal or approximately 25 kcal/kg per day. This amount of energy
is less than calculated requirements and reflects the limits posed by infusing a 10% dextrose
solution. However, based on patient tolerance, the infusion rate may be increased gradually to
enhance energy intake. A 1.5% amino acid infusion administered at the stated rate should
provide approximately 1 g/kg protein. Adding significantly greater amounts to the initial infusate
(eg, 3.0% amino acids) may result in excessive protein consumption if the rate is increased.
Excessive protein intake potentially can increase the risk for hyperammonemia and azotemia due
to protein catabolism.
Lipid infusions always should commence at a lower-than-targeted amount to allow for
metabolic adaptation to intravenous fat and to prevent hypertriglyceridemia. Indeed, for shortterm administration, 0.5 g/kg per day of intravenous lipids should be sufficient to prevent
deficiency of essential fatty acids. The decision to increase lipids beyond this level depends on
several factors, including the need to increase energy intake in a patient who has compromised
nutritional status and any fluid volume limitations.
Some have stated that intravenous lipid should be avoided in the patient who has
pancreatitis because hyperlipidemia is a known risk factor for pancreatitis. However, available
evidence demonstrates that appropriately monitored, lipid-based intravenous nutrition regimens
are well tolerated in affected patients.
References:
Kleinman RE. Parenteral nutrition. In: Pediatric Nutrition Handbook. 5th ed. Elk Grove Village, Ill:
American Academy of Pediatrics; 2004:369-389
Pietzak MM, Thomas DW. Pancreatitis in childhood. Pediatr Rev. 2000;21:406-412. Available at:
http://pedsinreview.aappublications.org/cgi/content/full/21/12/406
Shulman RJ, Phillips S. Parenteral nutrition indications, administration, and monitoring. In: Baker
SS, Baker RD, Davis AM, eds. Pediatric Nutrition Support. Sudbury, Mass: Jones and Bartlett
Publishers; 2006:273-286
Silberman H, Dixon NP, Eisenberg D. The safety and efficacy of a lipid-based system of
parenteral nutrition in acute pancreatitis. Am J Gastroenterol. 1982;77:494-497. Abstract
available at: http://www.ncbi.nlm.nih.gov/pubmed/6178287

Copyright 2010 by the American Academy of Pediatrics

page 24

Dr

_F

aq

eh

2010 PREP SA on CD-ROM


Question: 210

You are meeting with a pregnant woman who has received a liver transplant and is taking
chronic immunosuppression therapy. She asks you if the drugs she takes will preclude
breastfeeding her infant.
Of the following, the immunosuppressive drug that has the BEST safety profile for lactating
women is

A. cyclophosphamide
B. cyclosporin A
C. methotrexate
D. prednisone
E. tacrolimus

Copyright 2010 by the American Academy of Pediatrics

page 25

Dr

_F

aq

eh

2010 PREP SA on CD-ROM


Critique: 210

Preferred Response: D

Many medications prescribed to lactating mothers pose no risk for the newborn. Among
those for which there are concerns, antineoplastic agents and immunosuppressant drugs are
best known. The woman in the vignette, who has a history of solid organ transplantation, is
receiving immunosuppressant therapies. One of the safest immunosuppressive drugs is
prednisone (Item C210A).
When a physician or other health-care professional is uncertain of the potential
contraindication of a medication in a breastfeeding mother, he or she should contact the
pharmacist because many resources available to clinicians may, in fact, be out of date.
Cyclophosphamide and cyclosporin A both enter human milk and are transferred to the
nursing infant, but they have unknown effects on infant growth. Methotrexate is known to
cause neutropenia. Tacrolimus may enter the milk, and data are insufficient to declare it safe in
all instances. Other medications pose a high risk for breastfeeding infants (Item C210B).
As a result of reviewing this information, do you intend to make a change in practice
to provide better patient care?
Yes
No
References:
Akus M, Bartick M. Lactation safety recommendations and reliability compared in 10 medication
resources. Ann Pharmacother. 2007;41:1352-1360. Abstract available at:
http://www.ncbi.nlm.nih.gov/pubmed/17623757
American Academy of Pediatrics Committee on Drugs. The transfer of drugs and other
chemicals into human milk. Pediatrics. 2001;108:776-789. Available at:
http://pediatrics.aappublications.org/cgi/content/full/108/3/776
Coady NT. Maternal transplantation medications during breastfeeding. J Hum Lact. 2002;18:6668. Abstract available at: http://www.ncbi.nlm.nih.gov/pubmed/11845741
Gardiner SJ, Begg EJ. Breastfeeding during tacrolimus therapy. Obstet Gynecol. 2006;107:453455. Abstract available at: http://www.ncbi.nlm.nih.gov/pubmed/16449146
Hale TW. Pharmacology review: drug therapy and breastfeeding: antibiotics, analgesics, and
other medications. NeoReviews. 2005;6:e233-e240. Available for subscription at:
http://neoreviews.aappublications.org/cgi/content/full/6/5/e233
Hale TW. Pharmacology review: drug therapy and breastfeeding: antidepressants,
antipsychotics, antimanics, and sedatives. NeoReviews. 2004;5:e451-e456. Available for
subscription at: http://neoreviews.aappublications.org/cgi/content/full/5/10/e451
Mastrobattista JM, Gomez-Lobo V; Society for Maternal-Fetal Medicine. Pregnancy after solid

Copyright 2010 by the American Academy of Pediatrics

page 26

Dr

_F

aq

eh

2010 PREP SA on CD-ROM

organ transplantation. Obstet Gynecol. 2008;112:919-932. Abstract available at:


http://www.ncbi.nlm.nih.gov/pubmed/18827137

Copyright 2010 by the American Academy of Pediatrics

page 27

Dr

_F

aq

eh

2010 PREP SA on CD-ROM


Critique: 210

Copyright 2010 by the American Academy of Pediatrics

page 28

Dr

_F

aq

eh

2010 PREP SA on CD-ROM


Critique: 210

Copyright 2010 by the American Academy of Pediatrics

page 29

Dr

_F

aq

eh

2010 PREP SA on CD-ROM


Question: 226

A 30-weeks gestation very low-birthweight (VLBW) 1,400-g infant has respiratory distress
syndrome (RDS). He is receiving assisted ventilation following administration of three doses of
exogenous pulmonary surfactant. On his second postnatal day, his mother asks if she can
breastfeed her infant. You explain that she will not be able to breastfeed until he is extubated
and able to suckle. She asks you whether she should pump (express) her breast milk.
Of the following, the BEST response is that expressed human milk feedings

A. are contraindicated in VLBW infants who have RDS


B. are too difficult for VLBW infants to digest
C. can be fed by nasogastric tube
D. have no net benefit for VLBW infants once frozen
E. have too much protein for VLBW infants

Copyright 2010 by the American Academy of Pediatrics

page 30

Dr

_F

aq

eh

2010 PREP SA on CD-ROM


Critique: 226

Preferred Response: C

Preterm and very low-birthweight (VLBW) infants who have respiratory distress syndrome
(RDS) require medically assisted nutrition due not only to their illness but also because of their
neurologic immaturity. Oral suckling and coordinated respiration are developmentally challenging
until most preterm infants attain a postconceptual age of 32 to 34 weeks. Until that time, mothers
of such infants should be encouraged to express and store their milk for future use. When
deemed appropriate, generally by the third postnatal day, most VLBW preterm infants (including
those who have RDS) can be offered a small volume of milk feedings via nasogastric tube to
stimulate gastrointestinal function. Such small trophic or priming volumes (generally only 10
mL/kg per day total) are not relied upon for nutrition, which is provided parenterally.
Nonetheless, infants ultimately benefit from receiving human milk (even if frozen) through
enhanced immune system function, improved tolerance of feedings and ease of digestion, a
reduction in the incidence of necrotizing enterocolitis, and improved neurodevelopmental
outcomes and intelligence quotients.
Many preterm infants require supplementation to their mothers milk because preterm human
milk is low in total protein and certain minerals. In addition, those who have increased caloric
needs, due to RDS or chronic lung disease, may need supplementation of the expressed human
milk with fortifiers (liquid or solid). Human milk fortifiers provide additional protein, calories,
calcium, phosphorous and other minerals, and vitamins to meet the specific needs of preterm
infants. Iron supplements should be provided to preterm infants receiving only human milk once
they attain the goal volume of enteral feeding.
References:
Amaizu N, Shulman R, Schanler R, Lau C. Maturation of oral feeding skills in preterm infants. Acta
Paediatr. 2008;97:61-67. Abstract available at: http://www.ncbi.nlm.nih.gov/pubmed/18052999
Gewolb IH, Vice FL. Abnormalities in the coordination of respiration and swallow in preterm
infants with bronchopulmonary dysplasia. Dev Med Child Neurol. 2006;48:595-599. Abstract
available at: http://www.ncbi.nlm.nih.gov/pubmed/16780630
Nyqvist KH. Early attainment of breastfeeding competence in very preterm infants. Acta
Paediatr. 2008;97:776-781. Abstract available at:
http://www.ncbi.nlm.nih.gov/pubmed/18460108
Raab EL. The resuscitation & care of the newborn at risk. In: DeCherney AH, Nathan L, eds.
CURRENT Diagnosis & Treatment Obstetrics & Gynecology. 10th ed. New York, NY: The
McGraw-Hill Company, Inc; 2007:Chapter 31. Available for subscription at:
http://www.accessmedicine.com/content.aspx?aID=2387621
Thilo EH, Rosenberg AA. The newborn infant. In: Hay WW Jr, Levin MJ, Sondheimer JM,
Deterding RR, eds. CURRENT Diagnosis & Treatment: Pediatrics. 19th ed. New York, NY: The
McGraw-Hill Companies; 2009:Chapter 1. Available for subscription at:

Copyright 2010 by the American Academy of Pediatrics

page 31

Dr

_F

aq

eh

2010 PREP SA on CD-ROM

http://www.accessmedicine.com/content.aspx?aID=3396500
Vice FL, Gewolb IH. Respiratory patterns and strategies during feeding in preterm infants. Dev
Med Child Neurol. 2008;50:467-472. Abstract available at:
http://www.ncbi.nlm.nih.gov/pubmed/18422676

Copyright 2010 by the American Academy of Pediatrics

page 32

Vous aimerez peut-être aussi